320: Exam 4

Lakukan tugas rumah & ujian kamu dengan baik sekarang menggunakan Quizwiz!

21. A pregnant woman's biophysical profile score is 8. She asks the nurse to explain the results. The nurse's best response is: a. "The test results are within normal limits." b. "Immediate delivery by cesarean birth is being considered." c. "Further testing will be performed to determine the meaning of this score." d. "An obstetric specialist will evaluate the results of this profile and, within the next week, will inform you of your options regarding delivery."

a. "The test results are within normal limits."

3. The nurse sees a woman for the first time when she is 30 weeks pregnant. The woman has smoked throughout the pregnancy, and fundal height measurements now are suggestive of growth restriction in the fetus. In addition to ultrasound to measure fetal size, what other tool would be useful in confirming the diagnosis? a. Doppler blood flow analysis b. Contraction stress test (CST) c. Amniocentesis d. Daily fetal movement counts

a. Doppler blood flow analysis

10. When nurses help their expectant mothers assess the daily fetal movement counts, they should be aware that: a. Alcohol or cigarette smoke can irritate the fetus into greater activity. b. "Kick counts" should be taken every half hour and averaged every 6 hours, with every other 6-hour stretch off. c. The fetal alarm signal should go off when fetal movements stop entirely for 12 hours. d. Obese mothers familiar with their bodies can assess fetal movement as well as average-size women.

c. The fetal alarm signal should go off when fetal movements stop entirely for 12 hours.

What assessment should be performed to assess for hypertensive disorders?

1. BP 2. Edema 3. DTRs (refelcct balance bt cerebral cortex and spinal cord) 4. assess for ankle clonus 5. Presence of proteinuria 6. Evaluate for s & s of severe preeclampsia: headache, epigastric pain, RUQ, visual disturbances

Nursing interventions for severe gestational hypertension and preeclampsia with severe features

1. Bed rest with side rails up 2. darkened enviroment 3. Mg sulfate therapy 4. anti hypertensive medications

Nursing intervention for a pt with eclampsia during seizure

1. Priority care: keep airway patent: turn head to one side, place pillow under one shoulder if possible 2. Call for assistance. Do not leave bedside 3. Raise side rails and pad them with a folded blanket or pillow if possible 4. Observe and record convulsion activity (note time and duration)

A woman is undergoing a nipple-stimulated CST. She is having contractions that occur every 3 minutes. The fetal heart rate (FHR) has a baseline heart rate of approximately 120 beats per minute without any decelerations. What is the correct interpretation of this test? a. Negative b. Positive c. Satisfactory d. Unsatisfactory

A Adequate uterine activity necessary for a CST consists of three contractions in a 10-minute time frame. If no decelerations are observed in the FHR pattern with the contractions, then the findings are considered to be negative. A positive CST indicates the presence of repetitive late FHR decelerations. The terms satisfactory or unsatisfactory are not applicable

Which information is the highest priority for the nurse to comprehend regarding the BPP? a. BPP is an accurate indicator of impending fetal well-being. b. BPP is a compilation of health risk factors of the mother during the later stages of pregnancy. c. BPP consists of a Doppler blood flow analysis and an amniotic fluid index (AFI). d. BPP involves an invasive form of an ultrasonic examination.

A An abnormal BPP score is one indication that labor should be induced. The BPP evaluates the health of the fetus, requires many different measures, and is a noninvasive procedure

A 39-year-old primigravida woman believes that she is approximately 8 weeks pregnant, although she has had irregular menstrual periods all her life. She has a history of smoking approximately one pack of cigarettes a day; however, she tells the nurse that she is trying to cut down. Her laboratory data are within normal limits. What diagnostic technique would be useful at this time? a. Ultrasound examination b. Maternal serum alpha-fetoprotein (MSAFP) screening c. Amniocentesis d. Nonstress test (NST)

A An ultrasound examination could be performed to confirm the pregnancy and to determine the gestational age of the fetus. An MSAFP screening is performed at 16 to 18 weeks of gestation; therefore, it is too early in the womans pregnancy to perform this diagnostic test. An amniocentesis is performed if the MSAFP levels are abnormal or if fetal or maternal anomalies are detected. An NST is performed to assess fetal well-being in the third trimester.

Which analysis of maternal serum may predict chromosomal abnormalities in the fetus? a. Multiple-marker screening b. L/S ratio c. BPP d. Blood type and crossmatch of maternal and fetal serum

A Maternal serum can be analyzed for abnormal levels of alpha-fetoprotein, human chorionic gonadotropin, and estriol. The multiple-marker screening may predict chromosomal defects in the fetus. The L/S ratio is used to determine fetal lung maturity. A BPP is used for evaluating fetal status during the antepartum period. Five variables are used, but none is concerned with chromosomal problems. The blood type and crossmatch would not predict chromosomal defects in the fetus.

A woman with severe preeclampsia is receiving a magnesium sulfate infusion. The nurse becomes concerned after assessment when the woman exhibits A sleepy, sedated affect. A respiratory rate of 10 breaths/min. Deep tendon reflexes of 2+. Absence of ankle clonus.

A respiratory rate of 10 breaths/min. Rationale: A respiratory rate of 10 breaths/min indicates that the client is experiencing respiratory depression (bradypnea) from magnesium toxicity. Because magnesium sulfate is a central nervous system (CNS) depressant, the client will most likely become sedated when the infusion is initiated. Deep tendon reflexes of 2+ are a normal finding, as is absence of ankle clonus.

Which assessments are included in the fetal BPP? (Select all that apply.) a. Fetal movement b. Fetal tone c. Fetal heart rate d. AFI e. Placental grade

A, B, C, D Fetal movement, tone, heart rate, and AFI are all assessed in a BPP. The placental grade is determined by ultrasound and is not included in the criteria of assessment factors for a BPP.

A client has undergone an amniocentesis for evaluation of fetal well-being. Which intervention would be included in the nurse s plan of care after the procedure? (Select all that apply.) Select all that apply. A) Observe the client for possible uterine contractions. B) Perform a minicatheterization to obtain a urine specimen to assess for bleeding. C) Perform ultrasound to determine fetal positioning. D) Administer RhoGAM to the client if she is Rh negative.

A, D Ultrasound is used prior to the procedure as a visualization aid to assist with insertion of transabdominal needle. There is no need to assess the urine for bleeding as this is not considered to be a typical presentation or complication.

The corrected age of an infant who was born at 25 1/7 weeks and is preparing for discharge 124 days past delivery is ______________.

ANS: 42 6/7 weeks The age of a preterm newborn is corrected by adding the gestational age and the postnatal age. For example, an infant born at 32 weeks of gestation 4 weeks ago would now be considered 36 weeks of age. (32 + 4 = 36).

23. A client at 39 weeks of gestation has been admitted for an external version. Which intervention would the nurse anticipate the provider to order? a. Tocolytic drug b. Contraction stress test (CST) c. Local anesthetic d. Foley catheter

ANS: A A tocolytic drug will relax the uterus before and during the version, thus making manipulation easier. CST is used to determine the fetal response to stress. A local anesthetic is not used with external version. Although the bladder should be emptied, catheterization is not necessary. DIF: Cognitive Level: Apply REF: p. 779 TOP: Nursing Process: Planning MSC: Client Needs: Physiologic Integrity

18. What is the correct definition of a spontaneous termination of a pregnancy (abortion)? a. Pregnancy is less than 20 weeks. b. Fetus weighs less than 1000 g. c. Products of conception are passed intact. d. No evidence exists of intrauterine infection.

ANS: A An abortion is the termination of pregnancy before the age of viability (20 weeks). The weight of the fetus is not considered because some older fetuses may have a low birth weight. A spontaneous abortion may be complete or incomplete and may be caused by many problems, one being intrauterine infection. DIF: Cognitive Level: Remember REF: p. 669 TOP: Nursing Process: Assessment MSC: Client Needs: Health Promotion and Maintenance

4. A woman in preterm labor at 30 weeks of gestation receives two 12-mg intramuscular (IM) doses of betamethasone. What is the purpose of this pharmacologic intervention? a. To stimulate fetal surfactant production b. To reduce maternal and fetal tachycardia associated with ritodrine administration c. To suppress uterine contractions d. To maintain adequate maternal respiratory effort and ventilation during magnesium sulfate therapy

ANS: A Antenatal glucocorticoids administered as IM injections to the mother accelerate fetal lung maturity. Propranolol (Inderal) is given to reduce the effects of ritodrine administration. Betamethasone has no effect on uterine contractions. Calcium gluconate is given to reverse the respiratory depressive effects of magnesium sulfate therapy. DIF: Cognitive Level: Understand REF: p. 769 TOP: Nursing Process: Planning MSC: Client Needs: Physiologic Integrity

21. What is the highest priority nursing intervention when admitting a pregnant woman who has experienced a bleeding episode in late pregnancy? a. Assessing FHR and maternal vital signs b. Performing a venipuncture for hemoglobin and hematocrit levels c. Placing clean disposable pads to collect any drainage d. Monitoring uterine contractions

ANS: A Assessment of the FHR and maternal vital signs will assist the nurse in determining the degree of the blood loss and its effect on the mother and fetus. The most important assessment is to check the well-being of both the mother and the fetus. The blood levels can be obtained later. Assessing future bleeding is important; however, the top priority remains mother/fetal well-being. Monitoring uterine contractions is important but not a top priority. DIF: Cognitive Level: Apply REF: p. 681 TOP: Nursing Process: Implementation MSC: Client Needs: Health Promotion and Maintenance

Of these psychosocial factors, which has the least negative effect on the health of the mother and/or fetus? a. Moderate coffee consumption b. Moderate alcohol consumption c. Cigarette smoke d. Emotional distress

ANS: A Birth defects in humans have not been related to caffeine consumption. Pregnant women who consume more than 300 mg of caffeine daily may be at increased risk for miscarriage or intrauterine growth restriction (IUGR).

16. The exact cause of preterm labor is unknown but believed to be multifactorial. Infection is thought to be a major factor in many preterm labors. Which type of infection has not been linked to preterm birth? a. Viral b. Periodontal c. Cervical d. Urinary tract

ANS: A Infections that increase the risk of preterm labor and birth are bacterial and include cervical, urinary tract, periodontal, and other bacterial infections. Therefore, early, continual, and comprehensive participation by the client in her prenatal care is important. Recent evidence has shown a link between periodontal infections and preterm labor. Researchers recommend regular dental care before and during pregnancy, oral assessment as a routine part of prenatal care, and scrupulous oral hygiene to prevent periodontal infections. DIF: Cognitive Level: Remember REF: p. 760 TOP: Nursing Process: Assessment MSC: Client Needs: Physiologic Integrity

A pregnant woman was admitted for induction of labor at 43 weeks of gestation with sure dates. A nonstress test (NST) in the obstetrician's office revealed a nonreactive tracing. On artificial rupture of membranes, thick meconium-stained fluid was noted. What should the nurse caring for the infant after birth anticipate? a.Meconium aspiration, hypoglycemia, and dry, cracked skin b.Excessive vernix caseosa covering the skin, lethargy, and RDS c.Golden yellow to green-stained skin and nails, absence of scalp hair, and an increased amount of subcutaneous fat d.Hyperglycemia, hyperthermia, and an alert, wide-eyed appearance

ANS: A Meconium aspiration, hypoglycemia, and dry, cracked skin are consistent with a postmature infant. Excessive vernix caseosa, lethargy, and RDS are consistent with a very premature infant. The skin may be meconium stained, but the infant will most likely have long hair and decreased amounts of subcutaneous fat. Postmaturity with a nonreactive NST is indicative of hypoxia. Signs and symptoms associated with fetal hypoxia are hypoglycemia, temperature instability, and lethargy.

During the assessment of a preterm infant, the nurse notices continued respiratory distress even though oxygen and ventilation have been provided. In this situation, which condition should the nurse suspect? a.Hypovolemia and/or shock b.Excessively cool environment c.Central nervous system (CNS) injury d.Pending renal failure

ANS: A Other symptoms might include hypotension, prolonged capillary refill, and tachycardia, followed by bradycardia. Intervention is necessary. Preterm infants are susceptible to temperature instability. The goal of thermoregulation is to provide a neutral thermal environment. Hypoglycemia is likely to occur if the infant is attempting to conserve heat. CNS injury is manifested by hyperirritability, seizures, and abnormal movements of the extremities. Urine output and testing of specific gravity are appropriate interventions for the infant with suspected renal failure. This neonate is unlikely to be delivered with respiratory distress.

14. In caring for the woman with DIC, which order should the nurse anticipate? a. Administration of blood b. Preparation of the client for invasive hemodynamic monitoring c. Restriction of intravascular fluids d. Administration of steroids

ANS: A Primary medical management in all cases of DIC involves a correction of the underlying cause, volume replacement, blood component therapy, optimization of oxygenation and perfusion status, and continued reassessment of laboratory parameters. Central monitoring would not be initially ordered in a client with DIC because it could contribute to more areas of bleeding. Management of DIC would include volume replacement, not volume restriction. Steroids are not indicated for the management of DIC. DIF: Cognitive Level: Apply REF: pp. 685-686 TOP: Nursing Process: Planning MSC: Client Needs: Physiologic Integrity

9. With regard to hemorrhagic complications that may occur during pregnancy, what information is most accurate? a. An incompetent cervix is usually not diagnosed until the woman has lost one or two pregnancies. b. Incidences of ectopic pregnancy are declining as a result of improved diagnostic techniques. c. One ectopic pregnancy does not affect a woman's fertility or her likelihood of having a normal pregnancy the next time. d. Gestational trophoblastic neoplasia (GTN) is one of the persistently incurable gynecologic malignancies.

ANS: A Short labors and recurring losses of pregnancy at progressively earlier gestational ages are characteristics of reduced cervical competence. Because diagnostic technology is improving, more ectopic pregnancies are being diagnosed. One ectopic pregnancy places the woman at increased risk for another one. Ectopic pregnancy is a leading cause of infertility. Once invariably fatal, GTN now is the most curable gynecologic malignancy. DIF: Cognitive Level: Understand REF: p. 675 TOP: Nursing Process: Assessment MSC: Client Needs: Health Promotion and Maintenance

A premature infant with respiratory distress syndrome (RDS) receives artificial surfactant. How does the nurse explain surfactant therapy to the parents? a."Surfactant improves the ability of your baby's lungs to exchange oxygen and carbon dioxide." b."The drug keeps your baby from requiring too much sedation." c."Surfactant is used to reduce episodes of periodic apnea." d."Your baby needs this medication to fight a possible respiratory tract infection."

ANS: A Surfactant can be administered as an adjunct to oxygen and ventilation therapy. With the administration of an artificial surfactant, respiratory compliance is improved until the infant can generate enough surfactant on his or her own. Surfactant has no bearing on the sedation needs of the infant. Surfactant is used to improve respiratory compliance, including the exchange of oxygen and carbon dioxide. The goal of surfactant therapy in an infant with RDS is to stimulate the production of surfactant in the type 2 cells of the alveoli. The clinical presentation of RDS and neonatal pneumonia may be similar. The infant may be started on broad-spectrum antibiotics to treat infection.

13. In caring for an immediate postpartum client, the nurse notes petechiae and oozing from her intravenous (IV) site. The client would be closely monitored for which clotting disorder? a. DIC b. Amniotic fluid embolism (AFE) c. Hemorrhage d. HELLP syndrome

ANS: A The diagnosis of DIC is made according to clinical findings and laboratory markers. A physical examination reveals unusual bleeding. Petechiae may appear around a blood pressure cuff on the woman's arm. Excessive bleeding may occur from the site of slight trauma such as venipuncture sites. These symptoms are not associated with AFE, nor is AFE a bleeding disorder. Hemorrhage occurs for a variety of reasons in the postpartum client. These symptoms are associated with DIC. Hemorrhage would be a finding associated with DIC and is not a clotting disorder in and of itself. HELLP syndrome is not a clotting disorder, but it may contribute to the clotting disorder DIC. DIF: Cognitive Level: Understand REF: p. 685 TOP: Nursing Process: Planning MSC: Client Needs: Physiologic Integrity

What is the most important nursing action in preventing neonatal infection? a.Good handwashing b.Isolation of infected infants c.Separate gown technique d.Standard Precautions

ANS: A Virtually all controlled clinical trials have demonstrated that effective handwashing is responsible for the prevention of nosocomial infection in nursery units. Measures to be taken include Standard Precautions, careful and thorough cleaning, frequent replacement of used equipment, and disposal of excrement and linens in an appropriate manner. Overcrowding must be avoided in nurseries. However, the most important nursing action for preventing neonatal infection is effective handwashing.

Which risk factors are associated with NEC (Necrotizing enterocolitis)? (Select all that apply.) a.Polycythemia b.Anemia c.Congenital heart disease d.Bronchopulmonary dysphasia e.Retinopathy

ANS: A, B, C Risk factors for NEC include asphyxia, RDS, umbilical artery catheterization, exchange transfusion, early enteral feedings, patent ductus arteriosus (PDA), congenital heart disease, polycythemia, anemia, shock, and gastrointestinal infection. Bronchopulmonary dysphasia and retinopathy are not associated with NEC

Which risk factors are associated with NEC? (Select all that apply.) a.Polycythemia b.Anemia c.Congenital heart disease d.Bronchopulmonary dysphasia e.Retinopathy

ANS: A, B, C Risk factors for NEC include asphyxia, RDS, umbilical artery catheterization, exchange transfusion, early enteral feedings, patent ductus arteriosus (PDA), congenital heart disease, polycythemia, anemia, shock, and gastrointestinal infection. Bronchopulmonary dysphasia and retinopathy are not associated with NEC

2. What are the complications and risks associated with cesarean births? (Select all that apply.) a. Pulmonary edema b. Wound dehiscence c. Hemorrhage d. Urinary tract infections e. Fetal injuries

ANS: A, B, C, D, E Pulmonary edema, wound dehiscence, hemorrhage, urinary tract infections, and fetal injuries are possible complications and risks associated with cesarean births. DIF: Cognitive Level: Understand REF: p. 790 TOP: Nursing Process: Evaluation MSC: Client Needs: Physiologic Integrity

2. Approximately 10% to 15% of all clinically recognized pregnancies end in miscarriage. What are possible causes of early miscarriage? (Select all that apply.) a. Chromosomal abnormalities b. Infections c. Endocrine imbalance d. Systemic disorders e. Varicella

ANS: A, C, D, E Infections are not a common cause of early miscarriage. At least 50% of pregnancy losses result from chromosomal abnormalities. Endocrine imbalances such as hypothyroidism or diabetes are also possible causes for early pregnancy loss. Other systemic disorders that may contribute to pregnancy loss include lupus and genetic conditions. Although infections are not a common cause of early miscarriage, varicella infection in the first trimester has been associated with pregnancy loss. DIF: Cognitive Level: Remember REF: p. 669 TOP: Nursing Process: Assessment MSC: Client Needs: Health Promotion and Maintenance

MULTIPLE RESPONSE 1. Which adverse prenatal outcomes are associated with the HELLP syndrome? (Select all that apply.) a. Placental abruption b. Placenta previa c. Renal failure d. Cirrhosis e. Maternal and fetal death

ANS: A, C, E The HELLP syndrome is associated with an increased risk for adverse perinatal outcomes, including placental abruption, acute renal failure, subcapsular hepatic hematoma, hepatic rupture, recurrent preeclampsia, preterm birth, and fetal and maternal death. The HELLP syndrome is associated with an increased risk for placental abruption, not placenta previa. It is also associated with an increased risk for hepatic hematoma, not cirrhosis. DIF: Cognitive Level: Analyze REF: p. 658 TOP: Nursing Process: Assessment MSC: Client Needs: Physiologic Integrity

24. What is a maternal indication for the use of vacuum-assisted birth? a. Wide pelvic outlet b. Maternal exhaustion c. History of rapid deliveries d. Failure to progress past station 0

ANS: B A mother who is exhausted may be unable to assist with the expulsion of the fetus. The client with a wide pelvic outlet will likely not require vacuum extraction. With a rapid delivery, vacuum extraction is not necessary. A station of 0 is too high for a vacuum-assisted birth. DIF: Cognitive Level: Understand REF: p. 786 TOP: Nursing Process: Assessment MSC: Client Needs: Physiologic Integrity

18. The client being cared for has severe preeclampsia and is receiving a magnesium sulfate infusion. Which new finding would give the nurse cause for concern? a. Sleepy, sedated affect b. Respiratory rate of 10 breaths per minute c. DTRs of 2 d. Absent ankle clonus

ANS: B A respiratory rate of 10 breaths per minute indicates the client is experiencing respiratory depression from magnesium toxicity. Because magnesium sulfate is a CNS depressant, the client will most likely become sedated when the infusion is initiated. DTRs of 2 and absent ankle clonus are normal findings. DIF: Cognitive Level: Understand REF: p. 664 TOP: Nursing Process: Diagnosis MSC: Client Needs: Physiologic Integrity

14. A number of methods can be used for inducing labor. Which cervical ripening method falls under the category of mechanical or physical? a. Prostaglandins are used to soften and thin the cervix. b. Labor can sometimes be induced with balloon catheters or laminaria tents. c. Oxytocin is less expensive and more effective than prostaglandins but creates greater health risks. d. Amniotomy can be used to make the cervix more favorable for labor.

ANS: B Balloon catheters or laminaria tents are mechanical means of ripening the cervix. Ripening the cervix, making it softer and thinner, increases the success rate of induced labor. Prostaglandin E1 is less expensive and more effective than oxytocin but carries a greater risk. Amniotomy is the artificial rupture of membranes, which is used to induce labor only when the cervix is already ripe. DIF: Cognitive Level: Apply REF: p. 781 TOP: Nursing Process: Planning MSC: Client Needs: Health Promotion and Maintenance

In appraising the growth and development potential of a preterm infant, the nurse should be cognizant of the information that is best described in which statement? a.Tell the parents that their child will not catch up until approximately age 10 years (for girls) to age 12 years (for boys). b.Correct for milestones, such as motor competencies and vocalizations, until the child is approximately 2 years of age. c.Know that the greatest catch-up period is between 9 and 15 months postconceptual age. d.Know that the length and breadth of the trunk is the first part of the infant to experience catch-up growth.

ANS: B Corrections are made with a formula that adds gestational age and postnatal age. Whether a girl or boy, the infant experiences catch-up body growth during the first 2 to 3 years of life. Maximum catch-up growth occurs between 36 and 40 weeks of postconceptual age. The head is the first to experience catch-up growth.

11. Which laboratory marker is indicative of DIC? a. Bleeding time of 10 minutes b. Presence of fibrin split products c. Thrombocytopenia d. Hypofibrinogenemia

ANS: B Degradation of fibrin leads to the accumulation of multiple fibrin clots throughout the body's vasculature. Bleeding time in DIC is normal. Low platelets may occur but are not indicative of DIC because they may be the result from other coagulopathies. Hypofibrinogenemia occurs with DIC. DIF: Cognitive Level: Remember REF: p. 684 TOP: Nursing Process: Assessment MSC: Client Needs: Physiologic Integrity

17. Which maternal condition always necessitates delivery by cesarean birth? a. Marginal placenta previa b. Complete placenta previa c. Ectopic pregnancy d. Eclampsia

ANS: B In complete placenta previa, the placenta completely covers the cervical os. A cesarean birth is the acceptable method of delivery. The risk of fetal death occurring is due to preterm birth. If the previa is marginal (i.e., 2 cm or greater away from the cervical os), then labor can be attempted. A cesarean birth is not indicated for an ectopic pregnancy. Labor can be safely induced if the eclampsia is under control. DIF: Cognitive Level: Understand REF: p. 681 TOP: Nursing Process: Assessment MSC: Client Needs: Physiologic Integrity

19. What is the correct terminology for an abortion in which the fetus dies but is retained within the uterus? a. Inevitable abortion b. Missed abortion c. Incomplete abortion d. Threatened abortion

ANS: B Missed abortion refers to the retention of a dead fetus in the uterus. An inevitable abortion means that the cervix is dilating with the contractions. An incomplete abortion means that not all of the products of conception were expelled. With a threatened abortion, the woman has cramping and bleeding but no cervical dilation. DIF: Cognitive Level: Remember REF: p. 670 TOP: Nursing Process: Assessment MSC: Client Needs: Physiologic Integrity

16. In contrast to placenta previa, what is the most prevalent clinical manifestation of abruptio placentae? a. Bleeding b. Intense abdominal pain c. Uterine activity d. Cramping

ANS: B Pain is absent with placenta previa and may be agonizing with abruptio placentae. Bleeding may be present in varying degrees for both placental conditions. Uterine activity and cramping may be present with both placental conditions. DIF: Cognitive Level: Understand REF: p. 683 TOP: Nursing Process: Diagnosis MSC: Client Needs: Physiologic Integrity

In caring for the preterm infant, what complication is thought to be a result of high arterial blood oxygen level? a.NEC b.ROP c.BPD d.Intraventricular hemorrhage (IVH)

ANS: B ROP is thought to occur as a result of high levels of oxygen in the blood. NEC is caused by the interference of blood supply to the intestinal mucosa. Necrotic lesions occur at that site. BPD is caused by the use of positive pressure ventilation against the immature lung tissue. IVH results from the rupture of the fragile blood vessels in the ventricles of the brain and is most often associated with hypoxic injury, increased blood pressure, and fluctuating cerebral blood flow.

In the United States today: a. More than 20% of pregnancies meet the definition of high risk to either the mother or the infant b. Other than biophysical criteria, sociodemographic factors place both the mother and baby at risk c. High risk pregnancy status extends from first confirmation of pregnancy to birth d. High risk pregnancy is less critical a medical concern because of the reduction in family size and the decrease in unwanted pregnancies

ANS: B Sociodemographic risks include lack of prenatal care, low income, marital status, and ethnicity. Approximately 500,000 of the 4 million births (12.5%) in the United States are categorized as high risk. The high risk status for the mother extends through 30 days after childbirth. The reduction in family size and the decrease in unwanted pregnancies, along with technologic advances that facilitate pregnancies in previously infertile couples and advance the potential of care, have enhanced emphasis on delivering babies safely.

4. A 26-year-old pregnant woman, gravida 2, para 1-0-0-1, is 28 weeks pregnant when she experiences bright red, painless vaginal bleeding. On her arrival at the hospital, which diagnostic procedure will the client most likely have performed? a. Amniocentesis for fetal lung maturity b. Transvaginal ultrasound for placental location c. Contraction stress test (CST) d. Internal fetal monitoring

ANS: B The presence of painless bleeding should always alert the health care team to the possibility of placenta previa, which can be confirmed through ultrasonography. Amniocentesis is not performed on a woman who is experiencing bleeding. In the event of an imminent delivery, the fetus is presumed to have immature lungs at this gestational age, and the mother is given corticosteroids to aid in fetal lung maturity. A CST is not performed at a preterm gestational age. Furthermore, bleeding is a contraindication to a CST. Internal fetal monitoring is also contraindicated in the presence of bleeding. DIF: Cognitive Level: Apply REF: p. 680 TOP: Nursing Process: Assessment MSC: Client Needs: Health Promotion and Maintenance

8. A pregnant woman at 29 weeks of gestation has been diagnosed with preterm labor. Her labor is being controlled with tocolytic medications. She asks when she might be able to go home. Which response by the nurse is mostaccurate? a. "After the baby is born." b. "When we can stabilize your preterm labor and arrange home health visits." c. "Whenever your physician says that it is okay." d. "It depends on what kind of insurance coverage you have."

ANS: B This client's preterm labor is being controlled with tocolytics. Once she is stable, home care may be a viable option for this type of client. Care of a client with preterm labor is multidisciplinary and multifactorial; the goal is to prevent delivery. In many cases, this goal may be achieved at home. Managed care may dictate an earlier hospital discharge or a shift from hospital to home care. Insurance coverage may be one factor in client care, but ultimately, client safety remains the most important factor. DIF: Cognitive Level: Apply REF: p. 765 TOP: Nursing Process: Planning MSC: Client Needs: Health Promotion and Maintenance

5. Indications for a primary cesarean birth are often nonrecurring. Therefore, a woman who has had a cesarean birth with a low transverse scar may be a candidate for vaginal birth after cesarean (VBAC). Which clients would beless likely to have a successful VBAC? (Select all that apply.) a. Lengthy interpregnancy interval b. African-American race c. Delivery at a rural hospital d. Estimated fetal weight <4000 g e. Maternal obesity (BMI >30)

ANS: B, C, E Indications for a low success rate for a VBAC delivery include a short interpregnancy interval, non-Caucasian race, gestational age longer than 40 weeks, maternal obesity, preeclampsia, fetal weight greater than 4000 g, and delivery at a rural or private hospital. DIF: Cognitive Level: Understand REF: p. 794 TOP: Nursing Process: Assessment MSC: Client Needs: Physiologic Integrity

15. A woman arrives at the emergency department with complaints of bleeding and cramping. The initial nursing history is significant for a last menstrual period 6 weeks ago. On sterile speculum examination, the primary care provider finds that the cervix is closed. The anticipated plan of care for this woman would be based on a probable diagnosis of which type of spontaneous abortion? a. Incomplete b. Inevitable c. Threatened d. Septic

ANS: C A woman with a threatened abortion has spotting, mild cramps, and no cervical dilation. A woman with an incomplete abortion would have heavy bleeding, mild-to-severe cramping, and cervical dilation. An inevitable abortion demonstrates the same symptoms as an incomplete abortion: heavy bleeding, mild-to-severe cramping, and cervical dilation. A woman with a septic abortion has malodorous bleeding and typically a dilated cervix. DIF: Cognitive Level: Understand REF: p. 670 TOP: Nursing Process: Planning MSC: Client Needs: Physiologic Integrity

6. A woman with worsening preeclampsia is admitted to the hospital's labor and birth unit. The physician explains the plan of care for severe preeclampsia, including the induction of labor, to the woman and her husband. Which statement by the husband leads the nurse to believe that the couple needs further information? a. "I will help my wife use the breathing techniques that we learned in our childbirth classes." b. "I will give my wife ice chips to eat during labor." c. "Since we will be here for a while, I will call my mother so she can bring the two boys—2 years and 4 years of age—to visit their mother." d. "I will stay with my wife during her labor, just as we planned."

ANS: C Arranging a visit with their two children indicates that the husband does not understand the importance of the quiet, subdued environment that is needed to prevent his wife's condition from worsening. Implementing breathing techniques is indicative of adequate knowledge related to pain management during labor. Administering ice chips indicates an understanding of nutritional needs during labor. Staying with his wife during labor demonstrates the husband's support for his wife and is appropriate. DIF: Cognitive Level: Apply REF: p. 662 TOP: Nursing Process: Evaluation MSC: Client Needs: Psychosocial Integrity

3. The nurse is preparing to administer methotrexate to the client. This hazardous drug is most often used for which obstetric complication? a. Complete hydatidiform mole b. Missed abortion c. Unruptured ectopic pregnancy d. Abruptio placentae

ANS: C Methotrexate is an effective nonsurgical treatment option for a hemodynamically stable woman whose ectopic pregnancy is unruptured and measures less than 4 cm in diameter. Methotrexate is not indicated or recommended as a treatment option for a complete hydatidiform mole, for a missed abortion, or for abruptio placentae. DIF: Cognitive Level: Apply REF: p. 677 TOP: Nursing Process: Planning MSC: Client Needs: Physiologic Integrity

7. Prostaglandin gel has been ordered for a pregnant woman at 43 weeks of gestation. What is the primary purpose of prostaglandin administration? a. To enhance uteroplacental perfusion in an aging placenta b. To increase amniotic fluid volume c. To ripen the cervix in preparation for labor induction d. To stimulate the amniotic membranes to rupture

ANS: C Preparations of prostaglandin E1 and E2 are effective when used before labor induction to ripen (i.e., soften and thin) the cervix. Uteroplacental perfusion is not altered by the use of prostaglandins. The insertion of prostaglandin gel has no effect on the level of amniotic fluid. In some cases, women will spontaneously begin laboring after the administration of prostaglandins, thereby eliminating the need for oxytocin. It is not common for a woman's membranes to rupture as a result of prostaglandin use. DIF: Cognitive Level: Apply REF: p. 779 TOP: Nursing Process: Planning MSC: Client Needs: Physiologic Integrity

For clinical purposes, the most accurate definition of preterm and postterm infants is defined as what? a.Preterm: Before 34 weeks of gestation if the infant is appropriate for gestational age (AGA); before 37 weeks if the infant is small for gestational age (SGA) b.Postterm: After 40 weeks of gestation if the infant is large for gestational age (LGA); beyond 42 weeks if the infant is AGA c.Preterm: Before 37 weeks of gestation and postterm beyond 42 weeks of gestation; no matter the size for gestational age at birth d.Preterm: Before 38 to 40 weeks of gestation if the infant is SGA; postterm, beyond 40 to 42 weeks gestation if the infant is LGA

ANS: C Preterm and postterm are strictly measures of time—before 37 weeks and beyond 42 weeks, respectively—regardless of the size for gestational age.

An infant at 36 weeks of gestation has increasing respirations (80 to 100 breaths per minute with significant substernal retractions). The infant is given oxygen by continuous nasal positive airway pressure (CPAP). What level of partial pressure of arterial oxygen (PaO2) indicates hypoxia? a.67 mm Hg b.89 mm Hg c.45 mm Hg d.73 mm Hg

ANS: C The laboratory value of PaO2 of 45 mm Hg is below the range for a normal neonate and indicates hypoxia in this infant. The normal range for PaO2 is 60 to 80 mm Hg; therefore, PaO2 levels of 67 and 73 mm Hg fall within the normal range, and a PaO2 of 89 mm Hg is higher than the normal range.

6. A woman is having her first child. She has been in labor for 15 hours. A vaginal examination performed 2 hours earlier revealed the cervix to be dilated to 5 cm and 100% effaced, and the presenting part of the fetus was at station 0; however, another vaginal examination performed 5 minutes ago indicated no changes. What abnormal labor pattern is associated with this description? a. Prolonged latent phase b. Protracted active phase c. Secondary arrest d. Protracted descent

ANS: C With a secondary arrest of the active phase, the progress of labor has stopped. This client has not had any anticipated cervical change, indicating an arrest of labor. In the nulliparous woman, a prolonged latent phase typically lasts longer than 20 hours. A protracted active phase, the first or second stage of labor, is prolonged (slow dilation). With a protracted descent, the fetus fails to descend at an anticipated rate during the deceleration phase and second stage of labor. DIF: Cognitive Level: Analyze REF: p. 774 TOP: Nursing Process: Assessment MSC: Client Needs: Health Promotion and Maintenance

11. The nurse is performing an assessment on a client who thinks she may be experiencing preterm labor. Which information is the most important for the nurse to understand and share with the client? a. Because all women must be considered at risk for preterm labor and prediction is so variable, teaching pregnant women the symptoms of preterm labor probably causes more harm through false alarms. b. Braxton Hicks contractions often signal the onset of preterm labor. c. Because preterm labor is likely to be the start of an extended labor, a woman with symptoms can wait several hours before contacting the primary caregiver. d. Diagnosis of preterm labor is based on gestational age, uterine activity, and progressive cervical change.

ANS: D Gestational age of 20 to 37 weeks, uterine contractions, and a cervix that is 80% effaced or dilated 2 cm indicates preterm labor. It is essential that nurses teach women how to detect the early symptoms of preterm labor. Braxton Hicks contractions resemble preterm labor contractions, but they are not true labor. Waiting too long to see a health care provider could result in essential medications failing to be administered. Preterm labor is not necessarily long-term labor. DIF: Cognitive Level: Understand REF: p. 759 TOP: Nursing Process: Planning MSC: Client Needs: Safe and Effective Care Environment

7. The client has been on magnesium sulfate for 20 hours for the treatment of preeclampsia. She just delivered a viable infant girl 30 minutes ago. What uterine findings does the nurse expect to observe or assess in this client? a. Absence of uterine bleeding in the postpartum period b. Fundus firm below the level of the umbilicus c. Scant lochia flow d. Boggy uterus with heavy lochia flow

ANS: D High serum levels of magnesium can cause a relaxation of smooth muscle such as the uterus. Because of this tocolytic effect, the client will most likely have a boggy uterus with increased amounts of bleeding. All women experience uterine bleeding in the postpartum period, especially those who have received magnesium therapy. Rather than scant lochial flow, however, this client will most likely have a heavy flow attributable to the relaxation of the uterine wall caused by magnesium administration. DIF: Cognitive Level: Analyze REF: p. 664 TOP: Nursing Process: Assessment MSC: Client Needs: Physiologic Integrity

9. A woman is undergoing a nipple-stimulated contraction stress test (CST). She is having contractions that occur every 3 minutes. The fetal heart rate (FHR) has a baseline of approximately 120 beats/min without any decelerations. The interpretation of this test is said to be: a. Negative. b. Positive. c. Satisfactory. d. Unsatisfactory.

a. Negative.

With regard to maternal, fetal, and neonatal health problems, nurses should be aware that: a. Infection has replaced pulmonary embolism as one of the three top causes of maternal death attributable to pregnancy b. The leading cause of death in the neonatal period is disorders related to short gestation and low birth weight c. Factors related to the maternal death rate include age and marital status but not race d. Antepartum fetal deaths can best be prevented by better recognizing and responding to abnormalities of pregnancy and labor

ANS: D Medical teams need to be alert to signs of trouble. Race is a factor. African-American maternal mortality rates are more than three times higher than those for Caucasian women. Infection used to be an important cause of maternal death; it has been replaced by pulmonary embolism. The leading cause of death in the neonatal period is congenital anomalies. Race is a factor. African-American maternal mortality rates are more than three times higher than those for Caucasian women.

When evaluating the preterm infant, the nurse understands that compared with the term infant, what information is important for the nurse to understand? a.Few blood vessels visible through the skin b.More subcutaneous fat c.Well-developed flexor muscles d.Greater surface area in proportion to weight

ANS: D Preterm infants have greater surface area in proportion to their weight. More subcutaneous fat and well-developed muscles are indications of a more mature infant.

10. Which statement best describes chronic hypertension? a. Chronic hypertension is defined as hypertension that begins during pregnancy and lasts for the duration of the pregnancy. b. Chronic hypertension is considered severe when the systolic BP is higher than 140 mm Hg or the diastolic BP is higher than 90 mm Hg. c. Chronic hypertension is general hypertension plus proteinuria. d. Chronic hypertension can occur independently of or simultaneously with preeclampsia.

ANS: D Women with chronic hypertension may develop superimposed preeclampsia, which increases the morbidity for both the mother and the fetus. Chronic hypertension is present before pregnancy or diagnosed before the 20 weeks of gestation and persists longer than 6 weeks postpartum. Chronic hypertension becomes severe with a diastolic BP of 110 mm Hg or higher. Proteinuria is an excessive concentration of protein in the urine and is a complication of hypertension, not a defining characteristic. DIF: Cognitive Level: Understand REF: p. 667 TOP: Nursing Process: Diagnosis | Nursing Process: Planning MSC: Client Needs: Physiologic Integrity

Which assessment is not included in the fetal biophysical profile (BPP)? a. Fetal movement b. Fetal tone c. Fetal heart rate d. Amniotic fluid index e. Placental grade

ANS: E Fetal movement, tone, heart rate, and amniotic fluid index are all assessed in a BPP. The placental grade is determined by ultrasound (as is a BPP), but it is not included in the criteria of assessment factors for a BPP.

According to Beck's studies, what risk factor for postpartum depression is likely to have the greatest effect on the woman's condition? a. Prenatal depression b. Single-mother status c. Low socioeconomic status d. Unplanned or unwanted pregnancy

a. Prenatal depression Prenatal depression has been found by Beck to have the greatest likely effect. Single-mother status and low socioeconomic status are small-relation predictors, as is an unwanted pregnancy.

The perinatal nurse caring for the postpartum woman understands that late postpartum hemorrhage (PPH) is most likely caused by: a. Subinvolution of the placental site. b. Defective vascularity of the decidua. c. Cervical lacerations. d. Coagulation disorders.

a. Subinvolution of the placental site. Late PPH may be the result of subinvolution of the uterus, pelvic infection, or retained placental fragments. Late PPH is not typically a result of defective vascularity of the decidua, cervical lacerations, or coagulation disorders.

An MSAFP screening indicates an elevated level of alpha-fetoprotein. The test is repeated, and again the level is reported as higher than normal. What is the next step in the assessment sequence to determine the well-being of the fetus? a. PUBS b. Ultrasound for fetal anomalies c. BPP for fetal well-being d. Amniocentesis for genetic anomalies

B If MSAFP findings are abnormal, then follow-up procedures include genetic counseling for families with a history of NTD, repeated MSAFP screenings, an ultrasound examination, and possibly amniocentesis. Indications for the use of PUBS include prenatal diagnosis of inherited blood disorders, karyotyping of malformed fetuses, detection of fetal infection, determination of the acid-base status of fetuses with IUGR, and assessment and treatment of isoimmunization and thrombocytopenia in the fetus. A BPP is a method of assessing fetal well-being in the third trimester. Before an amniocentesis, the client would have an ultrasound for direct visualization of the fetus.

Which clinical finding is a major use of ultrasonography in the first trimester? a. Amniotic fluid volume b. Presence of maternal abnormalities c. Placental location and maturity d. Cervical length

B Ultrasonography can detect certain uterine abnormalities such as bicornuate uterus, fibroids, and ovarian cysts. Amniotic fluid volume, placental location and maturity, and cervical length are not available via ultrasonography until the second or third trimester

A nurse is reviewing assessments used to determine gestational age. When timeframe should the nurse identify as being the best to establish gestational age based on ultrasound? A) At term B) 36 weeks C) Between 14 and 22 weeks D) 8 weeks

C) Between 14 and 22 weeks Ultrasound determination of gestational age dating is best done between 14 and 22 weeks. It is less reliable after that period because of variability in fetal size. Standard sets of measurements relative to gestational age are noted around 10 to after 12 weeks and include crown-rump length (after 10), biparietal diameter (after 12), femur length, and head and abdominal circumferences.

A newborn in the neonatal intensive care unit (NICU) is dying as a result of a massive infection. The parents speak to the neonatologist, who informs them of their son's prognosis. When the father sees his son, he says, "He looks just fine to me. I can't understand what all this is about." The most appropriate response by the nurse would be: a. "Didn't the doctor tell you about your son's problems?" b. "This must be a difficult time for you. Tell me how you're doing." c. To stand beside him quietly. d. "You'll have to face up to the fact that he is going to die sooner or later."

b. "This must be a difficult time for you. Tell me how you're doing." The grief phase can be very difficult, especially for fathers. Parents should be encouraged to share their feelings as the initial steps in the grieving process. This father is in a phase of acute distress and is "reaching out" to the nurse as a source of direction in his grieving process. Shifting the focus is not in the best interest of the parent. Nursing actions may help the parents actualize the loss of their infant through sharing and verbalization of feelings of grief. "You'll have to face up to the fact that he is going to die sooner or later" is dispassionate and inappropriate statement.

What defines gestational hypertension as opposed to preeclampsia?

It is the onset of htn after week 20 but absent of protienuria and other diagnostics for preeclampsia > 140/90

Which of the following antihypertensive medications would cause a pregnant woman to have a positive Coombs test result? Nifedipine (Procardia) Methyldopa (Aldomet) Labetalol hydrochloride (Trandate) Hydralazine (Apresoline)

Methyldopa (Aldomet) A positive Coombs test result can occur in about 20% of patients taking methyldopa (Aldomet). None of the other drugs listed would have this effect.

With regard to preeclampsia and eclampsia, nurses should be aware that: Preeclampsia is a condition of the first trimester; eclampsia is a condition of the second and third trimesters. Preeclampsia results in decreased function in such organs as the placenta, kidneys, liver, and brain. The causes of preeclampsia and eclampsia are well documented. Severe preeclampsia is defined as preeclampsia plus proteinuria.

Preeclampsia results in decreased function in such organs as the placenta, kidneys, liver, and brain. Rationale: Vasospasms diminish the diameter of blood vessels, which impedes blood flow to all organs. Preeclampsia occurs after week 20 of gestation and can run the duration of the pregnancy. The causes of preeclampsia and eclampsia are unknown, although several have been suggested. Preeclampsia includes proteinuria; severe cases are characterized by greater proteinuria or any of nine other conditions.

What is preecampsia?

Pregnancy-specific condition in which hypertension and protienuria develop after week 20 in a normotensive pt

A woman delivered a 9-lb, 10-oz baby 1 hour ago. When you arrive to perform her 15-minute assessment, she tells you that she "feels all wet underneath." You discover that both pads are completely saturated and that she is lying in a 6-inch-diameter puddle of blood. What is your first action? a. Call for help. b.Assess the fundus for firmness. c.Take her blood pressure. d. Check the perineum for lacerations.

b. Assess the fundus for firmness.

4. A 41-week pregnant multigravida presents in the labor and delivery unit after a nonstress test indicated that her fetus could be experiencing some difficulties in utero. Which diagnostic tool would yield more detailed information about the fetus? a. Ultrasound for fetal anomalies b. Biophysical profile (BPP) c. Maternal serum alpha-fetoprotein (MSAFP) screening d. Percutaneous umbilical blood sampling (PUBS)

b. Biophysical profile (BPP)

A woman with severe preeclampsia is being treated with an IV infusion of magnesium sulfate. This treatment is considered successful if: Blood pressure is reduced to prepregnant baseline. Seizures do not occur. Deep tendon reflexes become hypotonic. Diuresis reduces fluid retention.

Seizures do not occur. Rationale: Magnesium sulfate is a central nervous system (CNS) depressant given primarily to prevent seizures. A temporary decrease in blood pressure can occur but is not the purpose of administering this medication. Hypotonia is a sign of an excessive serum level of magnesium. It is critical that calcium gluconate be on hand to counteract the depressant effects of magnesium toxicity. Diuresis is not an expected outcome of magnesium sulfate administration.

A steady trickle of bright red blood from the vagina in the presence of a firm fundus suggests: a. Uterine atony. b. Lacerations of the genital tract. c. Perineal hematoma. d. Infection of the uterus.

b. Lacerations of the genital tract.

A pregnant woman who is at 21 weeks of gestation has an elevated blood pressure of 140/98. Past medical history reveals that the woman has been treated for hypertension. On the basis of this information, the nurse would classify this patient as having: Preeclampsia. Gestational hypertension. Superimposed preeclampsia. Chronic hypertension.

Superimposed preeclampsia. Because this patient already has a medical history of hypertension and is now exhibiting hypertension after to the 20th week of gestation, she would be considered to have superimposed preeclampsia. Preeclampsia would be the classification in a patient without a history of hypertension who was hypertensive following the 20th week of pregnancy. Gestational hypertension occurs after the 20th week of pregnancy in a patient who was previously normotensive. Even though the patient has chronic hypertension, the fact that she is now pregnant determines that she would be classified as having superimposed preeclampsia.

Which instructions should be included in the discharge teaching plan to assist the patient in recognizing early signs of complications? a. Palpate the fundus daily to ensure that it is soft. b. Notify the physician of any increase in the amount of lochia or a return to bright red bleeding. c. Report any decrease in the amount of brownish red lochia. d. The passage of clots as large as an orange can be expected.

b. Notify the physician of any increase in the amount of lochia or a return to bright red bleeding.

7. Maternal serum alpha-fetoprotein (MSAFP) screening indicates an elevated level. MSAFP screening is repeated and again is reported as higher than normal. What would be the next step in the assessment sequence to determine the well-being of the fetus? a. Percutaneous umbilical blood sampling (PUBS) b. Ultrasound for fetal anomalies c. Biophysical profile (BPP) for fetal well-being d. Amniocentesis for genetic anomalies

b. Ultrasound for fetal anomalies

The nurse should be aware that a pessary would be most effective in the treatment of what disorder? a. Cystocele b. Uterine prolapse c. Rectocele d. Stress urinary incontinence

b. Uterine prolapse A fitted pessary may be inserted into the vagina to support the uterus and hold it in the correct position. A pessary is not used for a cystocele, a rectocele, or stress urinary incontinence.

12. In the first trimester, ultrasonography can be used to gain information on: a. Amniotic fluid volume. b. the presence of maternal abnormalities c. Placental location and maturity. d. Cervical length.

b. the presence of maternal abnormalities

Which options for saying goodbye would the nurse want to discuss with a woman who is diagnosed with having a stillborn girl? a. The nurse shouldn't discuss any options at this time; there is plenty of time after the baby is born. b. "Would you like a picture taken of your baby after birth?" c. "When your baby is born, would you like to see and hold her?" d. "What funeral home do you want notified after the baby is born?"

c. "When your baby is born, would you like to see and hold her?" Mothers and fathers may find it helpful to see the infant after delivery. The parents' wishes should be respected. Interventions and support from the nursing and medical staff after a prenatal loss are extremely important in the healing of the parents. Although this may be an intervention, the initial intervention should be related directly to the parents' wishes with regard to seeing or holding their dead infant. Although this information may be relevant, it is not the most appropriate option at this time. Burial arrangements can be discussed after the infant is born.

23. While working with the pregnant woman in her first trimester, the nurse is aware that chorionic villus sampling (CVS) can be performed during pregnancy at: a. 4 weeks b. 8 weeks c. 10 weeks d. 14 weeks

c. 10 weeks

14. With regard to amniocentesis, nurses should be aware that: a. Because of new imaging techniques, amniocentesis is now possible in the first trimester. b. Despite the use of ultrasound, complications still occur in the mother or infant in 5% to 10% of cases. c. Administration RhoD immunoglobulin may be necessary. d. The presence of meconium in the amniotic fluid is always cause for concern.

c. Administration RhoD immunoglobulin may be necessary.

5. At 35 weeks of pregnancy a woman experiences preterm labor. Tocolytics are administered and she is placed on bed rest, but she continues to experience regular uterine contractions, and her cervix is beginning to dilate and efface. What would be an important test for fetal well-being at this time? a. Percutaneous umbilical blood sampling (PUBS) b. Ultrasound for fetal size c. Amniocentesis for fetal lung maturity d. Nonstress test (NST)

c. Amniocentesis for fetal lung maturity

Complicated bereavement: a. Occurs when, in multiple births, one child dies, and the other or others live. b. Is a state in which the parents are ambivalent, as with an abortion. c. Is an extremely intense grief reaction that persists for a long time. d. Is felt by the family of adolescent mothers who lose their babies.

c. Is an extremely intense grief reaction that persists for a long time.

What is HELP syndrome?

a laboratory diagnosis for a variant of severe preeclampsia that involves hepatic dysfxn. It is not a separate illness

One of the first symptoms of puerperal infection to assess for in the postpartum woman is: a. Fatigue continuing for longer than 1 week. b. Pain with voiding. c. Profuse vaginal bleeding with ambulation. d. Temperature of 38° C (100.4° F) or higher on 2 successive days starting 24 hours after birth.

d. Temperature of 38° C (100.4° F) or higher on 2 successive days starting 24 hours after birth. Postpartum or puerperal infection is any clinical infection of the genital canal that occurs within 28 days after miscarriage, induced abortion, or childbirth. The definition used in the United States continues to be the presence of a fever of 38° C (100.4° F) or higher on 2 successive days of the first 10 postpartum days, starting 24 hours after birth. Fatigue would be a late finding associated with infection. Pain with voiding may indicate a urinary tract infection, but it is not typically one of the earlier symptoms of infection. Profuse lochia may be associated with endometritis, but it is not the first symptom associated with infection.

11. In comparing the abdominal and transvaginal methods of ultrasound examination, nurses should explain to their clients that: a. Both require the woman to have a full bladder. b. The abdominal examination is more useful in the first trimester. c. Initially the transvaginal examination can be painful. d. The transvaginal examination allows pelvic anatomy to be evaluated in greater detail.

d. The transvaginal examination allows pelvic anatomy to be evaluated in greater detail.

What non-specific clinical presentation might a pt report as a result of HELP syndrome?

hx of malaise, flu-like symptoms, epigastric/ RUQ pain, symptoms worse at night

Describe the pathophysiology of preeclampsia. What is the root cause? What is the main pathogenic factor?

*The placenta is the root cause* and, thus, the disease begins to resolve with the expelling of the placenta. Inadequate vascular remodeling in uterus = insufficient placental perfusion = endothelial cell dysfunction (release of substance toxic to endothelial cells) = This anomaly causes general vasospasm = poor tissue perfusion in all organ systems, increased peripheral resistance increased BP Increased endothelial cell permeability = loss of intravascular protein and fluid loss = less plasma volume = poor perfusion= further systemic effects Summary: *the main patho factor is not an increase in BP but poor perfusion as a result of vasospasm and reduced plasma volume*

Nursing interventions for mild gestational hypertension and preeclampsia without severe features

1. Activity restriction: complete or partial bed rest 2. maternal and fetal assessment 3. Diet: regular diet with adequate protien, calcium, folic acid, zinc, sodium, fluid intake

Nursing intervention for a pt with eclampsia after a siezure

1. Assess status of the woman's airway, breathing and pulse 2. Suction secretions from glottis to clear airway, 3. administer O2 4. Start IV if not in place or if previous infiltrated 5. Administer mg sulfate therapy

Goals of therapy for mild gestational hypertension and preeclampsia without severe features

1. Ensure maternal safety 2. Deliver newborn close to term

Goals of therapy for severe gestational hypertension and preeclampsia with severe features

1. Ensure maternal safety 2. formulate plan for delivery

Describe the diagnostic components/ criteria for preeclampsia

1. HTN: BP >140/90 X 2 at least 4 hrs apart 2. Protienuria: >300 mg in 24-hr specimen 3. Thrombocytopenia: platelet count < 1000,000/ uL 4. Impaired liver functions: elevated liver enzyme (transaminases) 5. Renal insufficiency: new development of serum creatinine > 1.1 mg/dL 6. Pulmonary edema 7. Cerebral/ visual disturbances

What are the systemic effects as a result of poor tissue perfusion and reduced plasma volume in a pt with preeclampsia?

1. Reduced kidney perfusion= decrease in glomerular filtration rate= protein (albumin) lost in urine, retaining of Na+ and H2O; necrosis and renal failure can occur 2. Decrease in plasma colloid osmotic pressure from decrease in levels of serum albumin= decrease in intravascular vol= hemoconcentration, increased blood viscosity, and tissue edema; increase in hct value = arteriolar vasospasm = endothelial damage and increased cap permeability= pulmonary edema 3. Decreased liver perfusion= impaired liver fxn and elevated liver enzyme levels; n&v, epigastric pain, RUQ pain 4. Cerebral edema and hemorrhage in increased CNS irritability (headaches, hyperflexia, clonus in ankle, seizures); arteriolar vasospasms and decreased blood flow to retina= visual disturbances

What is intrapartum nursing care directed toward for a pt diagnosed with severe gestational htn or preeclampsia with severe features?

1. early identification of fetal heart rate abnormalities 2. Prevention of maternal complications

What is HELP syndrome characterized by?

1. hemolysis 2. elevated liver enzymes 3. low platelet

Name four common risk factors for preeclampsia

1. primigravidity in woman <19 or >40 2. First pregnancy with new partner 3. Hx of preeclampsia 4. pregnancy-onset snoring

A client asks her nurse, My doctor told me that he is concerned with the grade of my placenta because I am overdue. What does that mean? What is the nurses best response? a. Your placenta changes as your pregnancy progresses, and it is given a score that indicates how well it is functioning. b. Your placenta isnt working properly, and your baby is in danger. c. We need to perform an amniocentesis to detect if you have any placental damage. d. Dont worry about it. Everything is fine.

A An explanation of what is meant by the grade of my placenta is the most appropriate response. If the client desires further information, the nurse can explain that calcium deposits are significant in postterm pregnancies, and ultrasonography can also be used to determine placental aging. Although stating that the clients placenta is not working properly and that the baby is in danger may be a valid response, it does not reflect therapeutic communication techniques and is likely to alarm the client. An ultrasound, not amniocentesis, is the method of assessment used to determine placental maturation. Telling the client not to worry is not appropriate and discredits her concerns.

Of these psychosocial factors, which has the least negative effect on the health of the mother and/or fetus? a. Moderate coffee consumption b. Moderate alcohol consumption c. Cigarette smoke d. Emotional distress

A Birth defects in humans have not been related to caffeine consumption. Pregnant women who consume more than 300 mg of caffeine daily may be at increased risk for miscarriage or IUGR. Although the exact effects of alcohol in pregnancy have not been quantified, it exerts adverse effects on the fetus including fetal alcohol syndrome, fetal alcohol effects, learning disabilities, and hyperactivity. A strong, consistent, causal relation has been established between maternal smoking and reduced birth weight. Childbearing triggers profound and complex physiologic and psychologic changes on the mother. Evidence suggests a relationship between emotional distress and birth complications.

The nurse sees a woman for the first time when she is 30 weeks pregnant. The client has smoked throughout the pregnancy, and fundal height measurements now are suggestive of intrauterine growth restriction (IUGR) in the fetus. In addition to ultrasound to measure fetal size, what is another tool useful in confirming the diagnosis? a. Doppler blood flow analysis b. Contraction stress test (CST) c. Amniocentesis d. Daily fetal movement counts

A Doppler blood flow analysis allows the examiner to study the blood flow noninvasively in the fetus and the placenta. It is a helpful tool in the management of high-risk pregnancies because of IUGR, diabetes mellitus, multiple fetuses, or preterm labor. Because of the potential risk of inducing labor and causing fetal distress, a CST is not performed on a woman whose fetus is preterm. Indications for an amniocentesis include diagnosis of genetic disorders or congenital anomalies, assessment of pulmonary maturity, and the diagnosis of fetal hemolytic disease, not IUGR. Fetal kick count monitoring is performed to monitor the fetus in pregnancies complicated by conditions that may affect fetal oxygenation. Although this may be a useful tool at some point later in this womans pregnancy, it is not used to diagnose IUGR.

Which information is an important consideration when comparing the CST with the NST? a. The NST has no known contraindications. b. The CST has fewer false-positive results when compared with the NST. c. The CST is more sensitive in detecting fetal compromise, as opposed to the NST. d. The CST is slightly more expensive than the NST.

A The CST has several contraindications. The NST has a high rate of false-positive results and is less sensitive than the CST but relatively inexpensive.

A pregnant womans BPP score is 8. She asks the nurse to explain the results. How should the nurse respond at this time? a. The test results are within normal limits. b. Immediate delivery by cesarean birth is being considered. c. Further testing will be performed to determine the meaning of this score. d. An obstetric specialist will evaluate the results of this profile and, within the next week, will inform you of your options regarding delivery.

A The normal biophysical score ranges from 8 to 10 points if the amniotic fluid volume is adequate. A normal score allows conservative treatment of high-risk clients. Delivery can be delayed if fetal well-being is indicated. Scores less than 4 should be investigated, and delivery could be initiated sooner than planned. The results of the BPP are usually available immediately after the procedure is performed. Since this score is within normal range, no further testing is required at this time.

26. Intrauterine growth restriction (IUGR) is associated with numerous pregnancy-related risk factors (Select all that apply). a. Poor nutrition b. Maternal collagen disease c. Gestational hypertension d. Premature rupture of membranes e. Smoking

A, B, C, E

27. Transvaginal ultrasonography is often performed during the first trimester. While preparing your 6-week gestation patient for this procedure, she expresses concerns over the necessity for this test. The nurse should explain that this diagnostic test may be indicated for a number of situations (Select all that apply). a. Multifetal gestation b. Obesity c. Fetal abnormalities d. Amniotic fluid volume e. Ectopic pregnancy

A, B, C, E

IUGR is associated with which pregnancy-related risk factors? (Select all that apply.) a. Poor nutrition b. Maternal collagen disease c. Gestational hypertension d. Premature rupture of membranes e. Smoking

A, B, C, E Poor nutrition, maternal collagen disease, gestational hypertension, and smoking are risk factors associated with the occurrence of IUGR. Premature rupture of membranes is associated with preterm labor, not IUGR.

Transvaginal ultrasonography is often performed during the first trimester. While preparing a 6-week gestational client for this procedure, she expresses concerns over the necessity for this test. The nurse should explain that this diagnostic test may be indicated for which situations? (Select all that apply.) a. Multifetal gestation b. Obesity c. Fetal abnormalities d. Amniotic fluid volume e. Ectopic pregnancy

A, B, C, E Transvaginal ultrasound is useful in women who are obese whose thick abdominal layers cannot be penetrated with traditional abdominal ultrasound. This procedure is also used to identify multifetal gestation, ectopic pregnancy, estimating gestational age, confirming fetal viability, and identifying fetal abnormalities. Amniotic fluid volume is assessed during the second and third trimester; conventional ultrasound would be used.

Medications used to manage postpartum hemorrhage (PPH) include (Select all that apply): a. Pitocin. b. Methergine. c.Terbutaline. d. Hemabate. e. Magnesium sulfate.

A, B, D Pitocin, Methergine, and Hemabate are all used to manage PPH. Terbutaline and magnesium sulfate are tocolytics; relaxation of the uterus causes or worsens PPH.

Cell-free deoxyribonucleic acid (DNA) screening is a new method of noninvasive prenatal testing (NIPT) that has recently become available in the clinical setting. This technology can provide a definitive diagnosis of which findings? (Select all that apply.) a. Fetal Rh status b. Fetal gender c. Maternally transmitted gene disorder d. Paternally transmitted gene disorder e. Trisomy 21

A, B, D, E The NIPT cannot actually distinguish fetal from maternal DNA. It can determine fetal Rh status, gender, trisomies 13, 18, and 21, as well as paternally transmitted gene disorders. The test can be performed any time after 10 weeks of gestation and is recommended for women who have previously given birth to a child with chromosomal abnormalities.

Possible alternative and complementary therapies for postpartum depression (PPD) for breastfeeding mothers include (Select all that apply): a. Acupressure. b. Aromatherapy. c. St. John's wort. d. Wine consumption. e. Yoga.

A, B, E Possible alternative/complementary therapies for postpartum depression include acupuncture, acupressure, aromatherapy, therapeutic touch, massage, relaxation techniques, reflexology, and yoga. St. John's wort has not been proven to be safe for women who are breastfeeding. Women who are breastfeeding and/or have a history of PPD should not consume alcohol.

12. When is a prophylactic cerclage for an incompetent cervix usually placed (in weeks of gestation)? a. 12 to 14 b. 6 to 8 c. 23 to 24 d. After 24

ANS: A A prophylactic cerclage is usually placed at 12 to 14 weeks of gestation. The cerclage is electively removed when the woman reaches 37 weeks of gestation or when her labor begins. Six to 8 weeks of gestation is too early to place the cerclage. Cerclage placement is offered if the cervical length falls to less than 20 to 25 mm before 23 to 24 weeks. Although no consensus has been reached, 24 weeks is used as the upper gestational age limit for cerclage placement. DIF: Cognitive Level: Apply REF: p. 674 TOP: Nursing Process: Planning MSC: Client Needs: Health Promotion and Maintenance

15. Which description most accurately describes the augmentation of labor? a. Is part of the active management of labor that is instituted when the labor process is unsatisfactory b. Relies on more invasive methods when oxytocin and amniotomy have failed c. Is a modern management term to cover up the negative connotations of forceps-assisted birth d. Uses vacuum cups

ANS: A Augmentation is part of the active management of labor that stimulates uterine contractions after labor has started but is not progressing satisfactorily. Augmentation uses amniotomy and oxytocin infusion, as well as some more gentle, noninvasive methods. Forceps-assisted births are less common than in the past and not considered a method of augmentation. A vacuum-assisted delivery occurs during childbirth if the mother is too exhausted to push. Vacuum extraction is not considered an augmentation methodology. DIF: Cognitive Level: Understand REF: pp. 785-786 TOP: Nursing Process: Planning MSC: Client Needs: Health Promotion and Maintenance

8. The client is being induced in response to worsening preeclampsia. She is also receiving magnesium sulfate. It appears that her labor has not become active, despite several hours of oxytocin administration. She asks the nurse, "Why is this taking so long?" What is the nurse's most appropriate response? a. "The magnesium is relaxing your uterus and competing with the oxytocin. It may increase the duration of your labor." b. "I don't know why it is taking so long." c. "The length of labor varies for different women." d. "Your baby is just being stubborn."

ANS: A Because magnesium sulfate is a tocolytic agent, its use may increase the duration of labor. The amount of oxytocin needed to stimulate labor may be more than that needed for the woman who is not receiving magnesium sulfate. The nurse should explain to the client the effects of magnesium sulfate on the duration of labor. Although the length of labor varies for different women, the most likely reason this woman's labor is protracted is the tocolytic effects of magnesium sulfate. The behavior of the fetus has no bearing on the length of labor. DIF: Cognitive Level: Apply REF: p. 664 TOP: Nursing Process: Planning MSC: Client Needs: Health Promotion and Maintenance

11. Which intervention is most important when planning care for a client with severe gestational hypertension? a. Induction of labor is likely, as near term as possible. b. If at home, the woman should be confined to her bed, even with mild gestational hypertension. c. Special diet low in protein and salt should be initiated. d. Vaginal birth is still an option, even in severe cases.

ANS: A By 34 weeks of gestation, the risk of continuing the pregnancy may be considered greater than the risks of a preterm birth. Strict bed rest is controversial for mild cases; some women in the hospital are even allowed to move around. Diet and fluid recommendations are essentially the same as for healthy pregnant women, although some authorities have suggested a diet high in protein. Women with severe gestational hypertension should expect a cesarean delivery. DIF: Cognitive Level: Apply REF: p. 660 TOP: Nursing Process: Planning MSC: Client Needs: Health Promotion and Maintenance

22. Which order should the nurse expect for a client admitted with a threatened abortion? a. Bed rest b. Administration of ritodrine IV c. Nothing by mouth (nil per os [NPO]) d. Narcotic analgesia every 3 hours, as needed

ANS: A Decreasing the woman's activity level may alleviate the bleeding and allow the pregnancy to continue. Ritodrine is not the first drug of choice for tocolytic medications. Having the woman placed on NPO is unnecessary. At times, dehydration may produce contractions; therefore, hydration is important. Narcotic analgesia will not decrease the contractions and may mask the severity of the contractions. DIF: Cognitive Level: Understand REF: pp. 671-672 TOP: Nursing Process: Planning MSC: Client Needs: Health Promotion and Maintenance

17. A woman with severe preeclampsia has been receiving magnesium sulfate by intravenous infusion for 8 hours. The nurse assesses the client and documents the following findings: temperature of 37.1° C, pulse rate of 96 beats per minute, respiratory rate of 24 breaths per minute, BP of 155/112 mm Hg, 3+ DTRs, and no ankle clonus. The nurse calls the provider with an update. The nurse should anticipate an order for which medication? a. Hydralazine b. Magnesium sulfate bolus c. Diazepam d. Calcium gluconate

ANS: A Hydralazine is an antihypertensive medication commonly used to treat hypertension in severe preeclampsia. Typically, it is administered for a systolic BP higher than 160 mm Hg or a diastolic BP higher than 110 mm Hg. An additional bolus of magnesium sulfate may be ordered for increasing signs of CNS irritability related to severe preeclampsia (e.g., clonus) or if eclampsia develops. Diazepam is sometimes used to stop or shorten eclamptic seizures. Calcium gluconate is used as the antidote for magnesium sulfate toxicity. The client is not currently displaying any signs or symptoms of magnesium toxicity. DIF: Cognitive Level: Analyze REF: p. 665 TOP: Nursing Process: Planning MSC: Client Needs: Physiologic Integrity

An infant at 26 weeks of gestation arrives intubated from the delivery room. The nurse weighs the infant, places him under the radiant warmer, and attaches him to the ventilator at the prescribed settings. A pulse oximeter and cardiorespiratory monitor are placed. The pulse oximeter is recording oxygen saturations of 80%. The prescribed saturations are 92%. What is the nurse's most appropriate action at this time? a.Listening to breath sounds, and ensuring the patency of the endotracheal tube, increasing oxygen, and notifying a physician b.Continuing to observe and making no changes until the saturations are 75% c.Continuing with the admission process to ensure that a thorough assessment is completed d.Notifying the parents that their infant is not doing well

ANS: A Listening to breath sounds and ensuring the patency of the endotracheal tube, increasing oxygen, and notifying a physician are appropriate nursing interventions to assist in optimal oxygen saturation of the infant. Oxygen saturation should be maintained above 92%, and oxygenation status of the infant is crucial. The nurse should delay other tasks to stabilize the infant. Notifying the parents that the infant is not doing well is not an appropriate action. Further assessment and intervention are warranted before determining fetal status.

15. Which neonatal complications are associated with hypertension in the mother? a. Intrauterine growth restriction (IUGR) and prematurity b. Seizures and cerebral hemorrhage c. Hepatic or renal dysfunction d. Placental abruption and DIC

ANS: A Neonatal complications are related to placental insufficiency and include IUGR, prematurity, and necrotizing enterocolitis. Seizures and cerebral hemorrhage are maternal complications. Hepatic and renal dysfunction are maternal complications of hypertensive disorders in pregnancy. Placental abruption and DIC are conditions related to maternal morbidity and mortality. DIF: Cognitive Level: Understand REF: p. 667 TOP: Nursing Process: Assessment MSC: Client Needs: Physiologic Integrity

19. What is the most common medical complication of pregnancy? a. Hypertension b. Hyperemesis gravidarum c. Hemorrhagic complications d. Infections

ANS: A Preeclampsia and eclampsia are two noted deadly forms of hypertension. A large percentage of pregnant women will have nausea and vomiting, but a relatively few will have the severe form called hyperemesis gravidarum. Hemorrhagic complications are the second most common medical complication of pregnancy; hypertension is the most common. Infection is a risk factor for preeclampsia. DIF: Cognitive Level: Remember REF: p. 653 TOP: Nursing Process: Assessment MSC: Client Needs: Physiologic Integrity

9. What nursing diagnosis is the most appropriate for a woman experiencing severe preeclampsia? a. Risk for injury to mother and fetus, related to central nervous system (CNS) irritability b. Risk for altered gas exchange c. Risk for deficient fluid volume, related to increased sodium retention secondary to the administration of magnesium sulfate d. Risk for increased cardiac output, related to the use of antihypertensive drugs

ANS: A Risk for injury is the most appropriate nursing diagnosis for this client scenario. Gas exchange is more likely to become impaired, attributable to pulmonary edema. A risk for excess, not deficient, fluid volume, related to increased sodium retention, is increased, and a risk for decreased, not increased, cardiac output, related to the use of antihypertensive drugs, also is increased. DIF: Cognitive Level: Apply REF: p. 660 TOP: Nursing Process: Diagnosis MSC: Client Needs: Physiologic Integrity

25. Which nursing intervention should be immediately performed after the forceps-assisted birth of an infant? a. Assessing the infant for signs of trauma b. Administering prophylactic antibiotic agents to the infant c. Applying a cold pack to the infant's scalp d. Measuring the circumference of the infant's head

ANS: A The infant should be assessed for bruising or abrasions at the site of application, facial palsy, and subdural hematoma. Prophylactic antibiotics are not necessary with a forceps delivery. A cold pack would place the infant at risk for cold stress and is contraindicated. Measuring the circumference of the head is part of the initial nursing assessment. DIF: Cognitive Level: Apply REF: p. 788 TOP: Nursing Process: Implementation MSC: Client Needs: Physiologic Integrity

20. A pregnant woman's amniotic membranes have ruptured. A prolapsed umbilical cord is suspected. What intervention would be the nurse's highest priority? a. Placing the woman in the knee-chest position b. Covering the cord in sterile gauze soaked in saline c. Preparing the woman for a cesarean birth d. Starting oxygen by face mask

ANS: A The woman is assisted into a modified Sims position, Trendelenburg position, or the knee-chest position in which gravity keeps the pressure of the presenting part off the cord. Although covering the cord in sterile gauze soaked saline, preparing the woman for a cesarean, and starting oxygen by face mark are appropriate nursing interventions in the event of a prolapsed cord, the intervention of top priority would be positioning the mother to relieve cord compression. DIF: Cognitive Level: Apply REF: p. 797 TOP: Nursing Process: Implementation MSC: Client Needs: Physiologic Integrity

3. Women who are obese are at risk for several complications during pregnancy and birth. Which of these would the nurse anticipate with an obese client? (Select all that apply.) a. Thromboembolism b. Cesarean birth c. Wound infection d. Breech presentation e. Hypertension

ANS: A, B, C, E A breech presentation is not a complication of pregnancy or birth for the client who is obese. Venous thromboembolism is a known risk for obese women. Therefore, the use of thromboembolism-deterrent (TED) hose and sequential compression devices may help decrease the chance for clot formation. Women should also be encouraged to ambulate as soon as possible. In addition to having an increased risk for complications with a cesarean birth, in general, obese women are also more likely to require an emergency cesarean birth. Many obese women have a pannus (i.e., large roll of abdominal fat) that overlies a lower transverse incision made just above the pubic area. The pannus causes the area to remain moist, which encourages the development of infection. Obese women are more likely to begin pregnancy with comorbidities such as hypertension and type 2 diabetes. DIF: Cognitive Level: Analyze REF: p. 778 TOP: Nursing Process: Assessment MSC: Client Needs: Health Promotion and Maintenance

3. The reported incidence of ectopic pregnancy has steadily risen over the past 2 decades. Causes include the increase in sexually transmitted infections (STIs) accompanied by tubal infection and damage. The popularity of contraceptive devices such as the IUD has also increased the risk for ectopic pregnancy. The nurse suspects that a client has early signs of ectopic pregnancy. The nurse should be observing the client for which signs or symptoms? (Select all that apply.) a. Pelvic pain b. Abdominal pain c. Unanticipated heavy bleeding d. Vaginal spotting or light bleeding e. Missed period

ANS: A, B, D, E A missed period or spotting can be easily mistaken by the client as an early sign of pregnancy. More subtle signs depend on exactly where the implantation occurs. The nurse must be thorough in her assessment because pain is not a normal symptom of early pregnancy. As the fallopian tube tears open and the embryo is expelled, the client often exhibits severe pain accompanied by intraabdominal hemorrhage, which may progress to hypovolemic shock with minimal or even no external bleeding. In approximately one half of women, shoulder and neck pain results from irritation of the diaphragm from the hemorrhage. DIF: Cognitive Level: Apply REF: p. 676 TOP: Nursing Process: Assessment MSC: Client Needs: Physiologic Integrity

2. One of the most important components of the physical assessment of the pregnant client is the determination of BP. Consistency in measurement techniques must be maintained to ensure that the nuances in the variations of the BP readings are not the result of provider error. Which techniques are important in obtaining accurate BP readings? (Select all that apply.) a. The client should be seated. b. The client's arm should be placed at the level of the heart. c. An electronic BP device should be used. d. The cuff should cover a minimum of 60% of the upper arm. e. The same arm should be used for every reading.

ANS: A, B, E BP readings are easily affected by maternal position. Ideally, the client should be seated. An alternative position is left lateral recumbent with the arm at the level of the heart. The arm should always be held in a horizontal position at approximately the level of the heart. The same arm should be used at every visit. The manual sphygmomanometer is the most accurate device. If manual and electronic devices are used in the care setting, then the nurse must use caution when interpreting the readings. A proper size cuff should cover at least 80% of the upper arm or be approximately 1.5 times the length of the upper arm. DIF: Cognitive Level: Apply REF: p. 658 TOP: Nursing Process: Assessment MSC: Client Needs: Physiologic Integrity

Infants born between 34 0/7 and 36 6/7 weeks of gestation are called late-preterm infants because they have many needs similar to those of preterm infants. Because they are more stable than early-preterm infants, they may receive care that is similar to that of a full-term baby. These infants are at increased risk for which conditions? (Select all that apply.) a.Problems with thermoregulation b.Cardiac distress c.Hyperbilirubinemia d.Sepsis e.Hyperglycemia

ANS: A, C, D Thermoregulation problems, hyperbilirubinemia, and sepsis are all conditions related to immaturity and warrant close observation. After discharge, the infant is at risk for rehospitalization related to these problems. Association of Women's Health, Obstetric and Neonatal Nurses (AWHONN) launched the Near-Term Infant Initiative to study the problem and ways to ensure that these infants receive adequate care. The nurse should ensure that this infant is adequately feeding before discharge and that parents are taught the signs and symptoms of these complications. Late-preterm infants are also at increased risk for respiratory distress and hypoglycemia.

4. The induction of labor is considered an acceptable obstetric procedure if it is in the best interest to deliver the fetus. The charge nurse on the labor and delivery unit is often asked to schedule clients for this procedure and therefore must be cognizant of the specific conditions appropriate for labor induction. What are appropriate indications for induction? (Select all that apply?) a. Rupture of membranes at or near term b. Convenience of the woman or her physician c. Chorioamnionitis (inflammation of the amniotic sac) d. Postterm pregnancy e. Fetal death

ANS: A, C, D, E The conditions listed are all acceptable indications for induction. Other conditions include intrauterine growth restriction (IUGR), maternal-fetal blood incompatibility, hypertension, and placental abruption. Elective inductions for the convenience of the woman or her provider are not recommended; however, they have become commonplace. Factors such as rapid labors and living a long distance from a health care facility may be valid reasons in such a circumstance. Elective delivery should not occur before 39 weeks of completed gestation. DIF: Cognitive Level: Apply REF: p. 790 TOP: Nursing Process: Planning MSC: Client Needs: Physiologic Integrity

MULTIPLE RESPONSE 1. A client who has undergone a D&C for early pregnancy loss is likely to be discharged the same day. The nurse must ensure that her vital signs are stable, that bleeding has been controlled, and that the woman has adequately recovered from the administration of anesthesia. To promote an optimal recovery, what information should discharge teaching include? (Select all that apply.) a. Iron supplementation b. Resumption of intercourse at 6 weeks postprocedure c. Referral to a support group, if necessary d. Expectation of heavy bleeding for at least 2 weeks e. Emphasizing the need for rest

ANS: A, C, E The woman should be advised to consume a diet high in iron and protein. For many women, iron supplementation also is necessary. The nurse should acknowledge that the client has experienced a loss, however early. She can be taught to expect mood swings and possibly depression. Referral to a support group, clergy, or professional counseling may be necessary. Discharge teaching should emphasize the need for rest. Nothing should be placed in the vagina for 2 weeks after the procedure, including tampons and vaginal intercourse. The purpose of this recommendation is to prevent infection. Should infection occur, antibiotics may be prescribed. The client should expect a scant, dark discharge for 1 to 2 weeks. Should heavy, profuse, or bright bleeding occur, she should be instructed to contact her health care provider. DIF: Cognitive Level: Apply REF: p. 672 TOP: Nursing Process: Implementation MSC: Client Needs: Physiologic Integrity

A premature infant never seems to sleep longer than an hour at a time. Each time a light is turned on, an incubator closes, or people talk near her crib, she wakes up and inconsolably cries until held. What is the correct nursing diagnosis beginning with "ineffective coping, related to"? a.Severe immaturity b.Environmental stress c.Physiologic distress d.Behavioral responses

ANS: B "Ineffective coping, related to environmental stress" is the most appropriate nursing diagnosis for this infant. Light and sound are known adverse stimuli that add to an already stressed premature infant. The nurse must closely monitor the environment for sources of overstimulation. Although the infant may be severely immature in this case, she is responding to environmental stress. Physiologic distress is the response to environmental stress. The result is stress cues such as increased metabolic rate, increased oxygen and caloric use, and depression of the immune system. The infant's behavioral response to the environmental stress is crying. The appropriate nursing diagnosis reflects the cause of this response.

NEC is an acute inflammatory disease of the gastrointestinal mucosa that can progress to perforation of the bowel. Approximately 2% to 5% of premature infants succumb to this fatal disease. Care is supportive; however, known interventions may decrease the risk of NEC. Which intervention has the greatest effect on lowering the risk of NEC? a.Early enteral feedings b.Breastfeeding c.Exchange transfusion d.Prophylactic probiotics

ANS: B A decrease in the incidence of NEC is directly correlated with exclusive breastfeeding. Breast milk enhances the maturation of the gastrointestinal tract and contains immune factors that contribute to a lower incidence or severity of NEC, Crohn disease, and celiac illness. The NICU nurse can be very supportive of the mother in terms of providing her with equipment to pump breast milk, ensuring privacy, and encouraging skin-to-skin contact with the infant. Early enteral feedings of formula or hyperosmolar feedings are a risk factor known to contribute to the development of NEC. The mother should be encouraged to pump or feed breast milk exclusively. Exchange transfusion may be necessary; however, it is a known risk factor for the development of NEC. Although still early, a study in 2005 found that the introduction of prophylactic probiotics appeared to enhance the normal flora of the bowel and therefore decrease the severity of NEC when it did occur. This treatment modality is not as widespread as encouraging breastfeeding; however, it is another strategy that the care providers of these extremely fragile infants may have at their disposal.

10. The nurse who elects to work in the specialty of obstetric care must have the ability to distinguish between preterm birth, preterm labor, and low birth weight. Which statement regarding this terminology is correct? a. Terms preterm birth and low birth weight can be used interchangeably. b. Preterm labor is defined as cervical changes and uterine contractions occurring between 20 and 37 weeks of gestation. c. Low birth weight is a newborn who weighs below 3.7 pounds. d. Preterm birth rate in the United States continues to increase.

ANS: B Before 20 weeks of gestation, the fetus is not viable (miscarriage); after 37 weeks, the fetus can be considered term. Although these terms are used interchangeably, they have different meanings: preterm birth describes the length of gestation (before 37 weeks), regardless of the newborn's weight; low birth weight describes only the infant's weight at the time of birth (2500 g or less), whenever it occurs. Low birth weight is anything below 2500 g or approximately pounds. In 2011, the preterm birth rate in the United States was 11.7 %; it has dropped every year since 2008. DIF: Cognitive Level: Understand REF: p. 759 TOP: Nursing Process: Assessment MSC: Client Needs: Health Promotion and Maintenance

2. A perinatal nurse is giving discharge instructions to a woman, status postsuction, and curettage secondary to a hydatidiform mole. The woman asks why she must take oral contraceptives for the next 12 months. What is the bestresponse by the nurse? a. "If you get pregnant within 1 year, the chance of a successful pregnancy is very small. Therefore, if you desire a future pregnancy, it would be better for you to use the most reliable method of contraception available." b. "The major risk to you after a molar pregnancy is a type of cancer that can be diagnosed only by measuring the same hormone that your body produces during pregnancy. If you were to get pregnant, then it would make the diagnosis of this cancer more difficult." c. "If you can avoid a pregnancy for the next year, the chance of developing a second molar pregnancy is rare. Therefore, to improve your chance of a successful pregnancy, not getting pregnant at this time is best." d. "Oral contraceptives are the only form of birth control that will prevent a recurrence of a molar pregnancy."

ANS: B Beta-human chorionic gonadotropin (beta-hCG) hormone levels are drawn for 1 year to ensure that the mole is completely gone. The chance of developing choriocarcinoma after the development of a hydatidiform mole is increased. Therefore, the goal is to achieve a zero human chorionic gonadotropin (hCG) level. If the woman were to become pregnant, then it may obscure the presence of the potentially carcinogenic cells. Women should be instructed to use birth control for 1 year after treatment for a hydatidiform mole. The rationale for avoiding pregnancy for 1 year is to ensure that carcinogenic cells are not present. Any contraceptive method except an intrauterine device (IUD) is acceptable. DIF: Cognitive Level: Apply REF: p. 679 TOP: Nursing Process: Planning | Nursing Process: Implementation MSC: Client Needs: Physiologic Integrity

With regard to infants who are SGA and intrauterine growth restriction (IUGR), the nurse should be aware of which information? a.In the first trimester, diseases or abnormalities result in asymmetric IUGR. b.Infants with asymmetric IUGR have the potential for normal growth and development. c.In asymmetric IUGR, weight is slightly larger than SGA, whereas length and head circumference are somewhat less than SGA. d.Symmetric IUGR occurs in the later stages of pregnancy.

ANS: B IUGR is either symmetric or asymmetric. The symmetric form occurs in the first trimester; infants who are SGA have reduced brain capacity. The asymmetric form occurs in the later stages of pregnancy. Weight is less than the 10th percentile; head circumference is greater than the 10th percentile. Infants with asymmetric IUGR have the potential for normal growth and development.

1. A pregnant woman is being discharged from the hospital after the placement of a cervical cerclage because of a history of recurrent pregnancy loss, secondary to an incompetent cervix. Which information regarding postprocedural care should the nurse emphasize in the discharge teaching? a. Any vaginal discharge should be immediately reported to her health care provider. b. The presence of any contractions, rupture of membranes (ROM), or severe perineal pressure should be reported. c. The client will need to make arrangements for care at home, because her activity level will be restricted. d. The client will be scheduled for a cesarean birth.

ANS: B Nursing care should stress the importance of monitoring for the signs and symptoms of preterm labor. Vaginal bleeding needs to be reported to her primary health care provider. Bed rest is an element of care. However, the woman may stand for periods of up to 90 minutes, which allows her the freedom to see her physician. Home uterine activity monitoring may be used to limit the woman's need for visits and to monitor her status safely at home. The cerclage can be removed at 37 weeks of gestation (to prepare for a vaginal birth), or a cesarean birth can be planned. DIF: Cognitive Level: Apply REF: p. 675 TOP: Nursing Process: Planning | Nursing Process: Implementation MSC: Client Needs: Health Promotion and Maintenance

Necrotizing enterocolitis (NEC) is an inflammatory disease of the gastrointestinal mucosa. The signs of NEC are nonspecific. What are generalized signs and symptoms of this condition? a.Hypertonia, tachycardia, and metabolic alkalosis b.Abdominal distention, temperature instability, and grossly bloody stools c.Hypertension, absence of apnea, and ruddy skin color d.Scaphoid abdomen, no residual with feedings, and increased urinary output

ANS: B Some generalized signs of NEC include decreased activity, hypotonia, pallor, recurrent apnea and bradycardia, decreased oxygen saturation values, respiratory distress, metabolic acidosis, oliguria, hypotension, decreased perfusion, temperature instability, cyanosis, abdominal distention, residual gastric aspirates, vomiting, grossly bloody stools, abdominal tenderness, and erythema of the abdominal wall. The infant may display hypotonia, bradycardia, and metabolic acidosis.

2. Which nursing intervention is paramount when providing care to a client with preterm labor who has received terbutaline? a. Assess deep tendon reflexes (DTRs). b. Assess for dyspnea and crackles. c. Assess for bradycardia. d. Assess for hypoglycemia.

ANS: B Terbutaline is a beta2-adrenergic agonist that affects the mother's cardiopulmonary and metabolic systems. Signs of cardiopulmonary decompensation include adventitious breath sounds and dyspnea. An assessment for dyspnea and crackles is important for the nurse to perform if the woman is taking magnesium sulfate. Assessing DTRs does not address the possible respiratory side effects of using terbutaline. Since terbutaline is a beta2-adrenergic agonist, it can lead to hyperglycemia, not hypoglycemia. Beta2-adrenergic agonist drugs cause tachycardia, not bradycardia. DIF: Cognitive Level: Analyze REF: pp. 767-768 TOP: Nursing Process: Assessment MSC: Client Needs: Physiologic Integrity

A newborn was admitted to the neonatal intensive care unit (NICU) after being delivered at 29 weeks of gestation to a 28-year-old multiparous, married, Caucasian woman whose pregnancy was uncomplicated until the premature rupture of membranes and preterm birth. The newborn's parents arrive for their first visit after the birth. The parents walk toward the bedside but remain approximately 5 feet away from the bed. What is the nurse's most appropriate action? a.Wait quietly at the newborn's bedside until the parents come closer. b.Go to the parents, introduce him or herself, and gently encourage them to meet their infant. Explain the equipment first, and then focus on the newborn. c.Leave the parents at the bedside while they are visiting so that they have some privacy. d.Tell the parents only about the newborn's physical condition and caution them to avoid touching their baby.

ANS: B The nurse is instrumental in the initial interactions with the infant. The nurse can help the parents see the infant rather than focus on the equipment. The importance and purpose of the apparatus that surrounds their infant also should be explained to them. Parents often need encouragement and recognition from the nurse to acknowledge the reality of the infant's condition. Parents need to see and touch their infant as soon as possible to acknowledge the reality of the birth and the infant's appearance and condition. Encouragement from the nurse is instrumental in this process. Telling the parents to avoid touching their baby is inappropriate and unhelpful.

5. A laboring woman with no known risk factors suddenly experiences spontaneous ROM. The fluid consists of bright red blood. Her contractions are consistent with her current stage of labor. No change in uterine resting tone has occurred. The fetal heart rate (FHR) begins to decline rapidly after the ROM. The nurse should suspect the possibility of what condition? a. Placenta previa b. Vasa previa c. Severe abruptio placentae d. Disseminated intravascular coagulation (DIC)

ANS: B Vasa previa is the result of a velamentous insertion of the umbilical cord. The umbilical vessels are not surrounded by Wharton jelly and have no supportive tissue. The umbilical blood vessels thus are at risk for laceration at any time, but laceration occurs most frequently during ROM. The sudden appearance of bright red blood at the time of ROM and a sudden change in the FHR without other known risk factors should immediately alert the nurse to the possibility of vasa previa. The presence of placenta previa most likely would be ascertained before labor and is considered a risk factor for this pregnancy. In addition, if the woman had a placenta previa, it is unlikely that she would be allowed to pursue labor and a vaginal birth. With the presence of severe abruptio placentae, the uterine tonicity typically is tetanus (i.e., a boardlike uterus). DIC is a pathologic form of diffuse clotting that consumes large amounts of clotting factors, causing widespread external bleeding, internal bleeding, or both. DIC is always a secondary diagnosis, often associated with obstetric risk factors such as the hemolysis, elevated liver enzyme levels, and low platelet levels (HELLP) syndrome. This woman did not have any prior risk factors. DIF: Cognitive Level: Analyze REF: p. 684 TOP: Nursing Process: Diagnosis MSC: Client Needs: Physiologic Integrity

MULTIPLE RESPONSE 1. The nurse recognizes that uterine hyperstimulation with oxytocin requires emergency interventions. What clinical cues alert the nurse that the woman is experiencing uterine hyperstimulation? (Select all that apply.) a. Uterine contractions lasting <90 seconds and occurring >2 minutes in frequency b. Uterine contractions lasting >90 seconds and occurring <2 minutes in frequency c. Uterine tone <20 mm Hg d. Uterine tone >20 mm Hg e. Increased uterine activity accompanied by a nonreassuring FHR and pattern

ANS: B, D, E Uterine contractions that occur less frequently than 2 minutes apart and last longer than 90 seconds, a uterine tone over 20 mm Hg, and a nonreassuring FHR and pattern are indications of uterine hyperstimulation with oxytocin administration. Uterine contractions that occur more frequently than 2 minutes apart and last less than 90 seconds are the expected goal of oxytocin induction. A uterine tone less than 20 mm Hg is normal. DIF: Cognitive Level: Analyze REF: p. 785 TOP: Nursing Process: Implementation MSC: Client Needs: Physiologic Integrity

21. What is the primary purpose for the use of tocolytic therapy to suppress uterine activity? a. Drugs can be efficaciously administered up to the designated beginning of term at 37 weeks gestation. b. Tocolytic therapy has no important maternal (as opposed to fetal) contraindications. c. The most important function of tocolytic therapy is to provide the opportunity to administer antenatal glucocorticoids. d. If the client develops pulmonary edema while receiving tocolytic therapy, then intravenous (IV) fluids should be given.

ANS: C Buying time for antenatal glucocorticoids to accelerate fetal lung development may be the best reason to use tocolytic therapy. Once the pregnancy has reached 34 weeks, however, the risks of tocolytic therapy outweigh the benefits. Important maternal contraindications to tocolytic therapy exist. Tocolytic-induced edema can be caused by IV fluids. DIF: Cognitive Level: Comprehend REF: p. 766 TOP: Nursing Process: Planning MSC: Client Needs: Physiologic Integrity

20. What condition indicates concealed hemorrhage when the client experiences abruptio placentae? a. Decrease in abdominal pain b. Bradycardia c. Hard, boardlike abdomen d. Decrease in fundal height

ANS: C Concealed hemorrhage occurs when the edges of the placenta do not separate. The formation of a hematoma behind the placenta and subsequent infiltration of the blood into the uterine muscle results in a very firm, boardlike abdomen. Abdominal pain may increase. The client will have shock symptoms that include tachycardia. As bleeding occurs, the fundal height increases. DIF: Cognitive Level: Analyze REF: p. 683 TOP: Nursing Process: Assessment MSC: Client Needs: Physiologic Integrity

6. A woman arrives for evaluation of signs and symptoms that include a missed period, adnexal fullness, tenderness, and dark red vaginal bleeding. On examination, the nurse notices an ecchymotic blueness around the woman's umbilicus. What does this finding indicate? a. Normal integumentary changes associated with pregnancy b. Turner sign associated with appendicitis c. Cullen sign associated with a ruptured ectopic pregnancy d. Chadwick sign associated with early pregnancy

ANS: C Cullen sign, the blue ecchymosis observed in the umbilical area, indicates hematoperitoneum associated with an undiagnosed ruptured intraabdominal ectopic pregnancy. Linea nigra on the abdomen is the normal integumentary change associated with pregnancy and exhibits a brown pigmented, vertical line on the lower abdomen. Turner sign is ecchymosis in the flank area, often associated with pancreatitis. A Chadwick sign is a blue-purple cervix that may be seen during or around the eighth week of pregnancy. DIF: Cognitive Level: Analyze REF: p. 676 TOP: Nursing Process: Assessment MSC: Client Needs: Physiologic Integrity

An infant is to receive gastrostomy feedings. Which intervention should the nurse institute to prevent bloating, gastrointestinal reflux into the esophagus, vomiting, and respiratory compromise? a.Rapid bolusing of the entire amount in 15 minutes b.Warm cloths to the abdomen for the first 10 minutes c.Slow, small, warm bolus feedings over 30 minutes d.Cold, medium bolus feedings over 20 minutes

ANS: C Feedings by gravity are slowly accomplished over 20- to 30-minute periods to prevent adverse reactions. Rapid bolusing would most likely lead to the adverse reactions listed. Temperature stability in the newborn is critical. Applying warm cloths to the abdomen would not be appropriate because the environment is not thermoregulated. In addition, abdominal warming is not indicated with feedings of any kind. Small feedings at room temperature are recommended to prevent adverse reactions.

2. The labor of a pregnant woman with preeclampsia is going to be induced. Before initiating the oxytocin (Pitocin) infusion, the nurse reviews the woman's latest laboratory test findings, which reveal a platelet count of 90,000 mm3, an elevated aspartate aminotransaminase (AST) level, and a falling hematocrit. The laboratory results are indicative of which condition? a. Eclampsia b. Disseminated intravascular coagulation (DIC) syndrome c. Hemolysis, elevated liver enzyme levels, and low platelet levels (HELLP) syndrome d. Idiopathic thrombocytopenia

ANS: C HELLP syndrome is a laboratory diagnosis for a variant of severe preeclampsia that involves hepatic dysfunction characterized by hemolysis (H), elevated liver (EL) enzymes, and low platelets (LP). Eclampsia is determined by the presence of seizures. DIC is a potential complication associated with HELLP syndrome. Idiopathic thrombocytopenia is the presence of low platelets of unknown cause and is not associated with preeclampsia. DIF: Cognitive Level: Understand REF: p. 657 TOP: Nursing Process: Diagnosis MSC: Client Needs: Physiologic Integrity

With regard to an eventual discharge of the high-risk newborn or the transfer of the newborn to a different facility, which information is essential to provide to the parents? a.Infants stay in the NICU until they are ready to go home. b.Once discharged to go home, the high-risk infant should be treated like any healthy term newborn. c.Parents of high-risk infants need special support and detailed contact information. d.If a high-risk infant and mother need to be transferred to a specialized regional center, then waiting until after the birth and until the infant is stabilized is best.

ANS: C High-risk infants can cause profound parental stress and emotional turmoil. Parents need support, special teaching, and quick access to various resources available to help them care for their baby. Parents and their high-risk infant should get to spend a night or two in a predischarge room, where care for the infant is provided away from the NICU. Simply because high-risk infants are eventually discharged does not mean they are normal, healthy babies. Follow-up by specialized practitioners is essential. Ideally, the mother and baby are transported with the fetus in utero; this reduces neonatal morbidity and mortality.

A nurse practicing in the perinatal setting should promote kangaroo care regardless of an infant's gestational age. Which statement regarding this intervention is most appropriate? a.Kangaroo care was adopted from classical British nursing traditions. b.This intervention helps infants with motor and CNS impairments. c.Kangaroo care helps infants interact directly with their parents and enhances their temperature regulation. d.This intervention gets infants ready for breastfeeding.

ANS: C Kangaroo care is skin-to-skin holding in which the infant, dressed only in a diaper, is placed directly on the parent's bare chest and then covered. The procedure helps infants interact with their parents and regulates their temperature, among other developmental benefits. Kangaroo care was established in Bogota, Colombia, assists the infant in maintaining an organized state, and decreases pain perception during heelsticks. Even premature infants who are unable to suckle benefit from kangaroo care. This practice fosters increased vigor and an enhanced breastfeeding experience as the infant matures.

12. What is the primary purpose for magnesium sulfate administration for clients with preeclampsia and eclampsia? a. To improve patellar reflexes and increase respiratory efficiency b. To shorten the duration of labor c. To prevent convulsions d. To prevent a boggy uterus and lessen lochial flow

ANS: C Magnesium sulfate is the drug of choice used to prevent convulsions, although it can generate other problems. Loss of patellar reflexes and respiratory depression are signs of magnesium toxicity. Magnesium sulfate can also increase the duration of labor. Women are at risk for a boggy uterus and heavy lochial flow as a result of magnesium sulfate therapy. DIF: Cognitive Level: Understand REF: p. 664 TOP: Nursing Process: Implementation MSC: Client Needs: Physiologic Integrity

14. Women with mild gestational hypertension and mild preeclampsia can be safely managed at home with frequent maternal and fetal evaluation. Complete or partial bed rest is still frequently ordered by some providers. Which complication is rarely the result of prolonged bed rest? a. Thrombophlebitis b. Psychologic stress c. Fluid retention d. Cardiovascular deconditioning

ANS: C No evidence has been found that supports the practice of bed rest to improve pregnancy outcome. Fluid retention is not an adverse outcome of prolonged bed rest. The woman is more likely to experience diuresis with accompanying fluid and electrolyte imbalance and weight loss. Prolonged bed rest is known to increase the risk for thrombophlebitis. Psychologic stress is known to begin on the first day of bed rest and continue for the duration of the therapy. Therefore, restricted activity, rather than complete bed rest, is recommended. Cardiovascular deconditioning is a known complication of bed rest. DIF: Cognitive Level: Understand REF: p. 661 TOP: Nursing Process: Diagnosis MSC: Client Needs: Physiologic Integrity

10. The management of the pregnant client who has experienced a pregnancy loss depends on the type of miscarriage and the signs and symptoms. While planning care for a client who desires outpatient management after a first-trimester loss, what would the nurse expect the plan to include? a. Dilation and curettage (D&C) b. Dilation and evacuation (D&E) c. Misoprostol d. Ergot products

ANS: C Outpatient management of a first-trimester loss is safely accomplished by the intravaginal use of misoprostol for up to 2 days. If the bleeding is uncontrollable, vital signs are unstable, or signs of infection are present, then a surgical evacuation should be performed. D&C is a surgical procedure that requires dilation of the cervix and scraping of the uterine walls to remove the contents of pregnancy. This procedure is commonly performed to treat inevitable or incomplete abortion and should be performed in a hospital. D&E is usually performed after 16 weeks of pregnancy. The cervix is widely dilated, followed by removal of the contents of the uterus. Ergot products such as Methergine or Hemabate may be administered for excessive bleeding after miscarriage. DIF: Cognitive Level: Apply REF: p. 672 TOP: Nursing Process: Planning MSC: Client Needs: Physiologic Integrity

1. In planning for home care of a woman with preterm labor, which concern should the nurse need to address? a. Nursing assessments are different from those performed in the hospital setting. b. Restricted activity and medications are necessary to prevent a recurrence of preterm labor. c. Prolonged bed rest may cause negative physiologic effects. d. Home health care providers are necessary.

ANS: C Prolonged bed rest may cause adverse effects such as weight loss, loss of appetite, muscle wasting, weakness, bone demineralization, decreased cardiac output, risk for thrombophlebitis, alteration in bowel functions, sleep disturbance, and prolonged postpartum recovery. Nursing assessments differ somewhat from those performed in the acute care setting, but this concern does not need to be addressed. Restricted activity and medications may prevent preterm labor but not in all women. In addition, the plan of care is individualized to meet the needs of each client. Many women receive home health nurse visits, but care is individualized for each woman. DIF: Cognitive Level: Analyze REF: p. 777 TOP: Nursing Process: Planning MSC: Client Needs: Health Promotion and Maintenance

1. A primigravida is being monitored at the prenatal clinic for preeclampsia. Which finding is of greatest concern to the nurse? a. Blood pressure (BP) increase to 138/86 mm Hg b. Weight gain of 0.5 kg during the past 2 weeks c. Dipstick value of 3+ for protein in her urine d. Pitting pedal edema at the end of the day

ANS: C Proteinuria is defined as a concentration of 1+ or greater via dipstick measurement. A dipstick value of 3+ alerts the nurse that additional testing or assessment should be performed. A 24-hour urine collection is preferred over dipstick testing attributable to accuracy. Generally, hypertension is defined as a BP of 140/90 mm Hg or an increase in systolic pressure of 30 mm Hg or diastolic pressure of 15 mm Hg. Preeclampsia may be demonstrated as a rapid weight gain of more than 2 kg in 1 week. Edema occurs in many normal pregnancies, as well as in women with preeclampsia. Therefore, the presence of edema is no longer considered diagnostic of preeclampsia. DIF: Cognitive Level: Analyze REF: p. 660 TOP: Nursing Process: Diagnosis MSC: Client Needs: Physiologic Integrity

4. A pregnant woman has been receiving a magnesium sulfate infusion for treatment of severe preeclampsia for 24 hours. On assessment, the nurse finds the following vital signs: temperature 37.3° C, pulse rate 88 beats per minute, respiratory rate 10 breaths per minute, BP 148/90 mm Hg, absent deep tendon reflexes (DTRs), and no ankle clonus. The client complains, "I'm so thirsty and warm." What is the nurse's immediate action? a. To call for an immediate magnesium sulfate level b. To administer oxygen c. To discontinue the magnesium sulfate infusion d. To prepare to administer hydralazine

ANS: C Regardless of the magnesium level, the client is displaying the clinical signs and symptoms of magnesium toxicity. The first action by the nurse should be to discontinue the infusion of magnesium sulfate. In addition, calcium gluconate, the antidote for magnesium, may be administered. Hydralazine is an antihypertensive drug commonly used to treat hypertension in severe preeclampsia. Typically, hydralazine is administered for a systolic BP higher than 160 mm Hg or a diastolic BP higher than 110 mm Hg. DIF: Cognitive Level: Apply REF: p. 664 TOP: Nursing Process: Implementation MSC: Client Needs: Physiologic Integrity

17. The nurse is teaching a client with preterm premature rupture of membranes (PPROM) regarding self-care activities. Which activities should the nurse include in her teaching? a. Report a temperature higher than 40° C. b. Tampons are safe to use to absorb the leaking amniotic fluid. c. Do not engage in sexual activity. d. Taking frequent tub baths is safe.

ANS: C Sexual activity should be avoided because it may induce preterm labor. A temperature higher than 38° C should be reported. To prevent the risk of infection, tub baths should be avoided and nothing should be inserted into the vagina. Further, foul-smelling vaginal fluid, which may be a sign of infection, should be reported. DIF: Cognitive Level: Apply REF: p. 762 TOP: Nursing Process: Implementation MSC: Client Needs: Health Promotion and Maintenance

8. A woman who is 30 weeks of gestation arrives at the hospital with bleeding. Which differential diagnosis would not be applicable for this client? a. Placenta previa b. Abruptio placentae c. Spontaneous abortion d. Cord insertion

ANS: C Spontaneous abortion is another name for miscarriage; it occurs, by definition, early in pregnancy. Placenta previa is a well-known reason for bleeding late in pregnancy. The premature separation of the placenta (abruptio placentae) is a bleeding disorder that can occur late in pregnancy. Cord insertion may cause a bleeding disorder that can also occur late in pregnancy. DIF: Cognitive Level: Understand REF: p. 669 TOP: Nursing Process: Assessment MSC: Client Needs: Physiologic Integrity, Physiologic Adaptation

20. Which statement most accurately describes the HELLP syndrome? a. Mild form of preeclampsia b. Diagnosed by a nurse alert to its symptoms c. Characterized by hemolysis, elevated liver enzymes, and low platelets d. Associated with preterm labor but not perinatal mortality

ANS: C The acronym HELLP stands for hemolysis (H), elevated liver (EL) enzymes, and low platelets (LP). The HELLP syndrome is a variant of severe preeclampsia and is difficult to identify because the symptoms are not often obvious. The HELLP syndrome must be diagnosed in the laboratory. Preterm labor is greatly increased; therefore, so is perinatal mortality. DIF: Cognitive Level: Understand REF: p. 657 TOP: Nursing Process: Diagnosis | Nursing Process: Planning MSC: Client Needs: Physiologic Integrity

An infant is being discharged from the NICU after 70 days of hospitalization. The infant was born at 30 weeks of gestation with several conditions associated with prematurity, including RDS, mild bronchopulmonary dysplasia (BPD), and retinopathy of prematurity (ROP), requiring surgical treatment. During discharge teaching, the infant's mother asks the nurse if her baby will meet developmental milestones on time, as did her son who was born at term. What is the nurse's most appropriate response? a."Your baby will develop exactly like your first child." b."Your baby does not appear to have any problems at this time." c."Your baby will need to be corrected for prematurity." d."Your baby will need to be followed very closely."

ANS: C The age of a preterm newborn is corrected by adding the gestational age and the postnatal age. The infant's responses are accordingly evaluated against the norm expected for the corrected age of the infant. The baby is currently 40 weeks of postconceptional age and can be expected to be doing what a 40-week-old infant would be doing. Although predicting with complete accuracy the growth and development potential of each preterm infant is impossible, certain measurable factors predict normal growth and development. The preterm infant experiences catch-up body growth during the first 2 to 3 years of life. Development needs to be evaluated over time. The growth and developmental milestones are corrected for gestational age until the child is approximately years old.

6.A 30-year-old gravida 3, para 2-0-0-2 is at 18 weeks of gestation. What screening test should be suggested to her? a. Biophysical profile b. Chorionic villi sampling c. Maternal serum alpha-fetoprotein (MSAFP) screening d. Screening for diabetes mellitus

ANS: C The biochemical assessment MSAFP test is performed from week 15 to week 20 of gestation (weeks 16 to 18 are ideal). A biophysical profile is a method of biophysical assessment of fetal well-being in the third trimester. Chorionic villi sampling is a biochemical assessment of the fetus that should be performed from the tenth to twelfth weeks of gestation. Screening for diabetes mellitus begins with the first prenatal visit.

5. A primigravida at 40 weeks of gestation is having uterine contractions every to 2 minutes and states that they are very painful. Her cervix is dilated 2 cm and has not changed in 3 hours. The woman is crying and wants an epidural. What is the likely status of this woman's labor? a. She is exhibiting hypotonic uterine dysfunction. b. She is experiencing a normal latent stage. c. She is exhibiting hypertonic uterine dysfunction. d. She is experiencing precipitous labor.

ANS: C The contraction pattern observed in this woman signifies hypertonic uterine activity. Typically, uterine activity in this phase occurs at 4- to 5-minute intervals lasting 30 to 45 seconds. Women who experience hypertonic uterine dysfunction, or primary dysfunctional labor, are often anxious first-time mothers who are having painful and frequent contractions that are ineffective at causing cervical dilation or effacement to progress. With hypotonic uterine dysfunction, the woman initially makes normal progress into the active stage of labor; then the contractions become weak and inefficient or stop altogether. Precipitous labor is one that lasts less than 3 hours from the onset of contractions until time of birth. DIF: Cognitive Level: Apply REF: p. 773 TOP: Nursing Process: Diagnosis MSC: Client Needs: Health Promotion and Maintenance

By understanding the four mechanisms of heat transfer (convection, conduction, radiation, and evaporation), the nurse can create an environment for the infant that prevents temperature instability. Which significant symptoms will the infant display when experiencing cold stress? a.Decreased respiratory rate b.Bradycardia, followed by an increased heart rate c.Mottled skin with acrocyanosis d.Increased physical activity

ANS: C The infant has minimal-to-no fat stores. During times of cold stress, the skin becomes mottled and acrocyanosis develops, progressing to cyanosis. Even if the infant is being cared for on a radiant warmer or in an isolette, the nurse's role is to observe the infant frequently to prevent heat loss and to respond quickly if signs and symptoms of cold stress occur. The respiratory rate increases, followed by periods of apnea. The infant initially tries to conserve heat and burns more calories, after which the metabolic system goes into overdrive. In the preterm infant who is experiencing heat loss, the heart rate initially increases, followed by periods of bradycardia. In the term infant, increased physical activity is the natural response to heat loss. However, in a term infant who is experiencing respiratory distress or in a preterm infant, physical activity is decreased.

23. Which finding on a prenatal visit at 10 weeks of gestation might suggest a hydatidiform mole? a. Complaint of frequent mild nausea b. Blood pressure of 120/80 mm Hg c. Fundal height measurement of 18 cm d. History of bright red spotting for 1 day, weeks ago

ANS: C The uterus in a hydatidiform molar pregnancy is often larger than would be expected on the basis of the duration of the pregnancy. Nausea increases in a molar pregnancy because of the increased production of hCG. A woman with a molar pregnancy may have early-onset pregnancy-induced hypertension. In the client's history, bleeding is normally described as brownish. DIF: Cognitive Level: Analyze REF: p. 678 TOP: Nursing Process: Assessment MSC: Client Needs: Health Promotion and Maintenance

13. The American College of Obstetricians and Gynecologists (ACOG) has developed a comprehensive list of risk factors associated with the development of preeclampsia. Which client exhibits the greatest number of these risk factors? a. 30-year-old obese Caucasian with her third pregnancy b. 41-year-old Caucasian primigravida c. 19-year-old African American who is pregnant with twins d. 25-year-old Asian American whose pregnancy is the result of donor insemination

ANS: C Three risk factors are present in the 19-year-old African-American client. She has African-American ethnicity, is at the young end of the age distribution, and has a multiple pregnancy. In planning care for this client, the nurse must frequently monitor her BP and teach her to recognize the early warning signs of preeclampsia. The 30-year-old obese Caucasian client has only has one known risk factor: obesity. Age distribution appears to be U-shaped, with women younger than 20 years of age and women older than 40 years of age being at greatest risk. Preeclampsia continues to be more frequently observed in primigravidas; this client is a multigravida woman. Two risk factors are present for the 41-year-old Caucasian primigravida client. Her age and status as a primigravida place her at increased risk for preeclampsia. Caucasian women are at a lower risk than are African-American women. The 25-year-old Asian-American client exhibits only one risk factor. Pregnancies that result from donor insemination, oocyte donation, and embryo donation are at an increased risk of developing preeclampsia. DIF: Cognitive Level: Analyze REF: p. 655 TOP: Nursing Process: Planning MSC: Client Needs: Physiologic Integrity

12. Which statement related to cephalopelvic disproportion (CPD) is the least accurate? a. CPD can be related to either fetal size or fetal position. b. The fetus cannot be born vaginally. c. CPD can be accurately predicted. d. Causes of CPD may have maternal or fetal origins.

ANS: C Unfortunately, accurately predicting CPD is not possible. Although CPD is often related to excessive fetal size (macrosomia), malposition of the fetal presenting part is the problem in many cases, not true CPD. When CPD is present, the fetus cannot fit through the maternal pelvis to be born vaginally. CPD may be related to either fetal origins such as macrosomia or malposition or maternal origins such as a too small or malformed pelvis. DIF: Cognitive Level: Understand REF: p. 775 TOP: Nursing Process: Planning MSC: Client Needs: Health Promotion and Maintenance

7. The nurse who elects to practice in the area of women's health must have a thorough understanding of miscarriage. Which statement regarding this condition is most accurate? a. A miscarriage is a natural pregnancy loss before labor begins. b. It occurs in fewer than 5% of all clinically recognized pregnancies. c. Careless maternal behavior, such as poor nutrition or excessive exercise, can be a factor in causing a miscarriage. d. If a miscarriage occurs before the 12th week of pregnancy, then it may be observed only as moderate discomfort and blood loss.

ANS: D Before the sixth week, the only evidence might be a heavy menstrual flow. After the 12th week, more severe pain, similar to that of labor, is likely. Miscarriage is a natural pregnancy loss, but it occurs, by definition, before 20 weeks of gestation, before the fetus is viable. Miscarriages occur in approximately 10% to 15% of all clinically recognized pregnancies. Miscarriages can be caused by a number of disorders or illnesses outside the mother's control or knowledge. DIF: Cognitive Level: Understand REF: p. 670 TOP: Nursing Process: Assessment MSC: Client Needs: Physiologic Integrity

Which condition might premature infants who exhibit 5 to 10 seconds of respiratory pauses, followed by 10 to 15 seconds of compensatory rapid respiration, be experiencing? a.Suffering from sleep or wakeful apnea b. Experiencing severe swings in blood pressure c.Trying to maintain a neutral thermal environment d.Breathing in a respiratory pattern common to premature infants

ANS: D Breathing in a respiratory pattern is called periodic breathing and is common to premature infants. This pattern may still require nursing intervention of oxygen and/or ventilation. Apnea is the cessation of respirations for 20 seconds or longer and should not be confused with periodic breathing.

18. A woman at 26 weeks of gestation is being assessed to determine whether she is experiencing preterm labor. Which finding indicates that preterm labor is occurring? a. Estriol is not found in maternal saliva. b. Irregular, mild uterine contractions are occurring every 12 to 15 minutes. c. Fetal fibronectin is present in vaginal secretions. d. The cervix is effacing and dilated to 2 cm.

ANS: D Cervical changes such as shortened endocervical length, effacement, and dilation are predictors of imminent preterm labor. Changes in the cervix accompanied by regular contractions indicate labor at any gestation. Estriol is a form of estrogen produced by the fetus that is present in plasma at 9 weeks of gestation. Levels of salivary estriol have been shown to increase before preterm birth. Irregular, mild contractions that do not cause cervical change are not considered a threat. The presence of fetal fibronectin in vaginal secretions between 24 and 36 weeks of gestation could predict preterm labor, but it has only a 20% to 40% positive predictive value. Of more importance are other physiologic clues of preterm labor such as cervical changes. DIF: Cognitive Level: Apply REF: p. 759 TOP: Nursing Process: Assessment | Nursing Process: Planning MSC: Client Needs: Health Promotion and Maintenance

Which clinical findings would alert the nurse that the neonate is expressing pain? a.Low-pitched crying; tachycardia; eyelids open wide b.Cry face; flaccid limbs; closed mouth c.High-pitched, shrill cry; withdrawal; change in heart rate d.Cry face; eyes squeezed; increase in blood pressure

ANS: D Crying and an increased heart rate are manifestations indicative of pain in the neonate. Typically, infants tightly close their eyes when in pain, not open them wide. In addition, infants may display a rigid posture with the mouth open and may also withdraw limbs and become tachycardic with pain. A high-pitched, shrill cry is associated with genetic or neurologic anomalies.

16. The nurse has evaluated a client with preeclampsia by assessing DTRs. The result is a grade of 3+. Which DTR response most accurately describes this score? a. Sluggish or diminished b. Brisk, hyperactive, with intermittent or transient clonus c. Active or expected response d. More brisk than expected, slightly hyperactive

ANS: D DTRs reflect the balance between the cerebral cortex and the spinal cord. They are evaluated at baseline and to detect changes. A slightly hyperactive and brisk response indicates a grade 3+ response. DIF: Cognitive Level: Apply REF: p. 660 TOP: Nursing Process: Assessment MSC: Client Needs: Physiologic Integrity

When providing an infant with a gavage feeding, which infant assessment should be documented each time? a.Abdominal circumference after the feeding b.Heart rate and respirations before feeding c.Suck and swallow coordination d.Response to the feeding

ANS: D Documentation of a gavage feeding should include the size of the feeding tube, the amount and quality of the residual from the previous feeding, the type and quantity of the fluid instilled, and the infant's response to the procedure. Abdominal circumference is not measured after a gavage feeding. Although vital signs may be obtained before feeding, the infant's response to the feeding is more important. Similarly, some older infants may be learning to suck; the most important factor to document would still be the infant's response to the feeding, including the attempts to suck.

3. A woman with preeclampsia has a seizure. What is the nurse's highest priority during a seizure? a. To insert an oral airway b. To suction the mouth to prevent aspiration c. To administer oxygen by mask d. To stay with the client and call for help

ANS: D If a client becomes eclamptic, then the nurse should stay with the client and call for help. Nursing actions during a convulsion are directed toward ensuring a patent airway and client safety. Insertion of an oral airway during seizure activity is no longer the standard of care. The nurse should attempt to keep the airway patent by turning the client's head to the side to prevent aspiration. Once the seizure has ended, it may be necessary to suction the client's mouth. Oxygen is administered after the convulsion has ended. DIF: Cognitive Level: Apply REF: p. 666 TOP: Nursing Process: Implementation MSC: Client Needs: Physiologic Integrity

13. Which statement related to the induction of labor is most accurate? a. Can be achieved by external and internal version techniques b. Is also known as a trial of labor (TOL) c. Is almost always performed for medical reasons d. Is rated for viability by a Bishop score

ANS: D Induction of labor is likely to be more successful with a Bishop score of 9 or higher for first-time mothers or 5 or higher for veterans. Version is the turning of the fetus to a better position by a physician for an easier or safer birth. A TOL is the observance of a woman and her fetus for several hours of active labor to assess the safety of vaginal birth. Two thirds of cases of induced labor are elective and not done for medical reasons. DIF: Cognitive Level: Understand REF: p. 780 TOP: Nursing Process: Diagnosis MSC: Client Needs: Safe and Effective Care Environment

22. When would an internal version be indicated to manipulate the fetus into a vertex position? a. Fetus from a breech to a cephalic presentation before labor begins b. Fetus from a transverse lie to a longitudinal lie before a cesarean birth c. Second twin from an oblique lie to a transverse lie before labor begins d. Second twin from a transverse lie to a breech presentation during a vaginal birth

ANS: D Internal version is used only during a vaginal birth to manipulate the second twin into a presentation that allows it to be vaginally born. For internal version to occur, the cervix needs to be completely dilated. DIF: Cognitive Level: Remember REF: p. 779 TOP: Nursing Process: Assessment MSC: Client Needs: Physiologic Integrity

On day 3 of life, a newborn continues to require 100% oxygen by nasal cannula. The parents ask if they may hold their infant during his next gavage feeding. Considering that this newborn is physiologically stable, what response should the nurse provide? a."Parents are not allowed to hold their infants who are dependent on oxygen." b."You may only hold your baby's hand during the feeding." c."Feedings cause more physiologic stress; therefore, the baby must be closely monitored. I don't think you should hold the baby." d."You may hold your baby during the feeding."

ANS: D Physical contact with the infant is important to establish early bonding. The nurse as the support person and teacher is responsible for shaping the environment and making the caregiving responsive to the needs of both the parents and the infant. Allowing the parents to hold their baby is the most appropriate response by the nurse. Parental interaction by holding should be encouraged during gavage feedings; nasal cannula oxygen therapy allows for easy feedings and psychosocial interactions. The parent can swaddle the infant or provide kangaroo care while gavage feeding their infant. Both swaddling and kangaroo care during feedings provide positive interactions for the infant and help the infant associate feedings with positive interactions.

19. Which assessment is least likely to be associated with a breech presentation? a. Meconium-stained amniotic fluid b. Fetal heart tones heard at or above the maternal umbilicus c. Preterm labor and birth d. Postterm gestation

ANS: D Postterm gestation is not likely to occur with a breech presentation. The presence of meconium in a breech presentation may be a result of pressure on the fetal wall as it traverses the birth canal. Fetal heart tones heard at the level of the umbilical level of the mother are a typical finding in a breech presentation because the fetal back would be located in the upper abdominal area. Breech presentations often occur in preterm births. DIF: Cognitive Level: Analyze REF: pp. 775-776 TOP: Nursing Process: Assessment MSC: Client Needs: Health Promotion and Maintenance

24. A 32-year-old primigravida is admitted with a diagnosis of ectopic pregnancy. Which information assists the nurse in developing the plan of care? a. Bed rest and analgesics are the recommended treatment. b. She will be unable to conceive in the future. c. A D&C will be performed to remove the products of conception. d. Hemorrhage is the primary concern.

ANS: D Severe bleeding occurs if the fallopian tube ruptures. The recommended treatment is to remove the pregnancy before rupture to prevent hemorrhaging. If the tube must be removed, then the woman's fertility will decrease; however, she will not be infertile. A D&C is performed on the inside of the uterine cavity. The ectopic pregnancy is located within the tubes. DIF: Cognitive Level: Apply REF: p. 676 TOP: Nursing Process: Planning MSC: Client Needs: Physiologic Integrity

9. The obstetric provider has informed the nurse that she will be performing an amniotomy on the client to induce labor. What is the nurse's highest priority intervention after the amniotomy is performed? a. Applying clean linens under the woman b. Taking the client's vital signs c. Performing a vaginal examination d. Assessing the fetal heart rate (FHR)

ANS: D The FHR is assessed before and immediately after the amniotomy to detect any changes that might indicate cord compression or prolapse. Providing comfort measures, such as clean linens, for the client is important but not the priority immediately after an amniotomy. The woman's temperature should be checked every 2 hours after the rupture of membranes but not the priority immediately after an amniotomy. The woman would have had a vaginal examination during the procedure. Unless cord prolapse is suspected, another vaginal examination is not warranted. Additionally, FHR assessment provides clinical cues to a prolapsed cord. DIF: Cognitive Level: Analyze REF: p. 783 TOP: Nursing Process: Implementation MSC: Client Needs: Physiologic Integrity

Because of the premature infant's decreased immune functioning, what nursing diagnosis should the nurse include in a plan of care for a premature infant? a.Delayed growth and development b.Ineffective thermoregulation c.Ineffective infant feeding pattern d.Risk for infection

ANS: D The nurse needs to understand that decreased immune functioning increases the risk for infection. Growth and development, thermoregulation, and feeding may be affected, although only indirectly.

3. In evaluating the effectiveness of magnesium sulfate for the treatment of preterm labor, which finding alerts the nurse to possible side effects? a. Urine output of 160 ml in 4 hours b. DTRs 2+ and no clonus c. Respiratory rate (RR) of 16 breaths per minute d. Serum magnesium level of 10 mg/dl

ANS: D The therapeutic range for magnesium sulfate management is 4 to 7.5 mg/dl. A serum magnesium level of 10 mg/dl could lead to signs and symptoms of magnesium toxicity, including oliguria and respiratory distress. Urine output of 160 ml in 4 hours, DTRs of 2+, and a RR of 16 breaths per minute are all normal findings. DIF: Cognitive Level: Apply REF: p. 767 TOP: Nursing Process: Evaluation MSC: Client Needs: Physiologic Integrity

5. A woman at 39 weeks of gestation with a history of preeclampsia is admitted to the labor and birth unit. She suddenly experiences increased contraction frequency of every 1 to 2 minutes, dark red vaginal bleeding, and a tense, painful abdomen. Which clinical change does the nurse anticipate? a. Eclamptic seizure b. Rupture of the uterus c. Placenta previa d. Abruptio placentae

ANS: D Uterine tenderness in the presence of increasing tone may be the earliest sign of abruptio placentae. Women with preeclampsia are at increased risk for an abruption attributable to decreased placental perfusion. Eclamptic seizures are evidenced by the presence of generalized tonic-clonic convulsions. Uterine rupture exhibits hypotonic uterine activity, signs of hypovolemia, and, in many cases, the absence of pain. Placenta previa exhibits bright red, painless vaginal bleeding. DIF: Cognitive Level: Understand REF: p. 662 TOP: Nursing Process: Diagnosis MSC: Client Needs: Physiologic Integrity

What it essential for early detection of htn disorders in a pregnant woman?

Accurate measure of BP

The nurse is planning the care for a laboring client with diabetes mellitus. This client is at greater risk for which clinical finding? a. Oligohydramnios b. Polyhydramnios c. Postterm pregnancy d. Chromosomal abnormalities

B Polyhydramnios or amniotic fluid in excess of 2000 ml is 10 times more likely to occur in the client with diabetes mellitus rather than in nondiabetic pregnancies. This complication places the mother at risk for premature rupture of membranes, premature labor, and postpartum hemorrhage. Prolonged rupture of membranes, IUGR, intrauterine fetal death, and renal agenesis (Potter syndrome) place the client at risk for developing oligohydramnios. Anencephaly, placental insufficiency, and perinatal hypoxia contribute to the risk for postterm pregnancy. Maternal age older than 35 years and balanced translocation (maternal and paternal) are risk factors for chromosomal abnormalities.

A 41-week pregnant multigravida arrives at the labor and delivery unit after a NST indicated that her fetus could be experiencing some difficulties in utero. Which diagnostic tool yields more detailed information about the condition of the fetus? a. Ultrasound for fetal anomalies b. Biophysical profile (BPP) c. MSAFP screening d. Percutaneous umbilical blood sampling (PUBS)

B Real-time ultrasound permits a detailed assessment of the physical and physiologic characteristics of the developing fetus and a cataloging of normal and abnormal biophysical responses to stimuli. The BPP is a noninvasive, dynamic assessment of a fetus that is based on acute and chronic markers of fetal disease. An ultrasound for fetal anomalies would most likely have occurred earlier in the pregnancy. It is too late in the pregnancy to perform an MSAFP. Furthermore, it does not provide information related to fetal well-being. Indications for PUBS include prenatal diagnosis or inherited blood disorders, karyotyping of malformed fetuses, detection of fetal infection, determination of the acid-base status of the fetus with IUGR, and assessment and treatment of isoimmunization and thrombocytopenia in the fetus.

Which nursing intervention is necessary before a first-trimester transabdominal ultrasound? a. Place the woman on nothing by mouth (nil per os [NPO]) for 12 hours. b. Instruct the woman to drink 1 to 2 quarts of water. c. Administer an enema. d. Perform an abdominal preparation.

B When the uterus is still in the pelvis, visualization may be difficult. Performing a first-trimester transabdominal ultrasound requires the woman to have a full bladder, which will elevate the uterus upward and provide a better visualization of the fetus; therefore, being NPO is not appropriate. Neither an enema nor an abdominal preparation is necessary for this procedure.

A nurse providing care for an antepartum woman receiving a contraction stress test (CST). Which statement should the nurse identify as being accurate? A) Sometimes uses vibroacoustic stimulation. B) Is considered to have a negative result if no late decelerations are observed with the contractions. C) Is more effective than nonstress test (NST) if the membranes have already been ruptured. D) Is an invasive test; however, contractions are stimulated.

B) Is considered to have a negative result if no late decelerations are observed with the contractions. No late decelerations indicate a positive CST result. Vibroacoustic stimulation is sometimes used with NST. CST is invasive if stimulation is performed by IV oxytocin but not if by nipple stimulation. CST is contraindicated if the membranes have ruptured.

A 40-year-old woman with a high body mass index (BMI) is 10 weeks pregnant. Which diagnostic tool should the nurse identify as being appropriate to assess the pregnancy? A) Biophysical profile B) Transvaginal ultrasound C) Amniocentesis D) Maternal serum alpha-fetoprotein (MSAFP)

B) Transvaginal ultrasound An ultrasound is the method of biophysical assessment of the infant that is performed at this gestational age. Transvaginal ultrasound is especially useful for obese women, whose thick abdominal layers cannot be penetrated adequately with the abdominal approach. A biophysical profile is a method of biophysical assessment of fetal well-being in the third trimester. An amniocentesis is performed after the fourteenth week of pregnancy. A MSAFP test is performed from week 15 to week 22 of the gestation (weeks 16 to 18 are ideal).

A nurse is providing instruction for an obstetrical client to perform a daily fetal movement count (DFMC). Which instructions should the nurse include in the plan of care? (Select all that apply.) A) The fetal alarm signal is reached when there are no fetal movements noted for 5 hours. B) Count all fetal movements in a 12-hour period daily until 10 fetal movements are noted. C) Monitor fetal activity two times a day either after meals or before bed for a period of 2 hours or until 10 fetal movements are noted. D) The client can monitor fetal activity once daily for a 60-minute period and note activity.

B, C, D DFMC is a non-invasive monitoring assessment that can be easily performed by the client. It represents no added financial or physical burden to the client and does not interfere in any daily routine. The client records fetal movements and typically documents them on a log. There are different protocols that can be used in terms of time frames but the primary focus is to establish a pattern of 10 movements. Absence of fetal movements in a twelve hour period would signal a fetal alarm. Fewer than 3 movements in an hour would warrant further investigation.

The nurse would conclude that grieving parents had progressed to the reorganization/recovery phase during a follow-up visit a year later if: a. The parents say they feel no pain. b. The parents are discussing sex and a future pregnancy, even if they have not sorted out their feelings yet. c. The parents have abandoned those moments of bittersweet grief. d. The parents' questions have progressed from "Why?" to "Why us?"

B. The parents are discussing sex and a future Many couples have conflicting feelings about sexuality and future pregnancies. A little pain is always present, certainly past the first year, when recovery begins to peak. Bittersweet grief describes the brief grief response that occurs with reminders of a loss, often on anniversary dates. Most couples never abandon it. Recovery is ongoing. Typically a couple's search for meaning progresses from "Why?" in the acute phase to "Why me?" in the intense phase to "What does this loss mean to my life?" in the reorganizational phase.

In the past, factors to determine whether a woman was likely to develop a high-risk pregnancy were primarily evaluated from a medical point of view. A broader, more comprehensive approach to high-risk pregnancy has been adopted today. Four categories have now been established, based on the threats to the health of the woman and the outcome of pregnancy. Which category should not be included in this group? a. Biophysical b. Psychosocial c. Geographic d. Environmental

C A geographic category is correctly referred to as sociodemographic risk. These factors stem from the mother and her family. Ethnicity may be one of the risks to pregnancy; however, it is not the only factor in this category. Low income, lack of prenatal care, age, parity, and marital status also are included. Biophysical is one of the broad categories used for determining risk. These include genetic considerations, nutritional status, and medical and obstetric disorders. Psychosocial risks include smoking, caffeine, drugs, alcohol, and psychologic status. All of these adverse lifestyles can have a negative effect on the health of the mother or fetus. Environmental risks are risks that can affect both fertility and fetal development. These include infections, chemicals, radiation, pesticides, illicit drugs, and industrial pollutants

At 35 weeks of pregnancy, a woman experiences preterm labor. Although tocolytic medications are administered and she is placed on bed rest, she continues to experience regular uterine contractions and her cervix is beginning to dilate and efface. What is an important test for fetal well-being at this time? a. PUBS b. Ultrasound for fetal size c. Amniocentesis for fetal lung maturity d. NST

C Amniocentesis is performed to assess fetal lung maturity in the event of a preterm birth. The fluid is examined to determine the lecithin to sphingomyelin (L/S) ratio. Indications for PUBS include prenatal diagnosis or inherited blood disorders, karyotyping of malformed fetuses, detection of fetal infection, determination of the acid-base status of the fetus with IUGR, and assessment and treatment of isoimmunization and thrombocytopenia in the fetus. Determination of fetal size by ultrasound is typically performed during the second trimester and is not indicated in this scenario. An NST measures the fetal response to fetal movement in a noncontracting mother.

A client in the third trimester has just undergone an amniocentesis to determine fetal lung maturity. Which statement regarding this testing is important for the nurse in formulating a care plan? a. Because of new imaging techniques, an amniocentesis should have been performed in the first trimester. b. Despite the use of ultrasonography, complications still occur in the mother or infant in 5% to 10% of cases. c. Administration of Rho(D) immunoglobulin may be necessary. d. The presence of meconium in the amniotic fluid is always a cause for concern.

C As a result of the possibility of fetomaternal hemorrhage, administration of Rho(D) immunoglobulin is the standard of practice after amniocentesis for women who are Rh negative. Amniocentesis is possible after the 14th week of pregnancy when the uterus becomes an abdominal organ. Complications occur in less than 1% of cases; many have been minimized or eliminated through the use of ultrasonography. Meconium in the amniotic fluid before the beginning of labor is not usually a problem.

While working with the pregnant client in her first trimester, what information does the nurse provide regarding when CVS can be performed (in weeks of gestation)? a. 4 b. 8 c. 10 d. 14

C CVS can be performed in the first or second trimester, ideally between 10 and 13 weeks of gestation. During this procedure, a small piece of tissue is removed from the fetal portion of the placenta. If performed after 9 completed weeks of gestation, then the risk of limb reduction is no greater than in the general population.

The indirect Coombs test is a screening tool for Rh incompatibility. If the titer is greater than ______, amniocentesis may be a necessary next step. a. 1:2 b. 1:4 c. 1:8 d. 1:12

C If the maternal titer for Rh antibodies is greater 1:8, then an amniocentesis is indicated to determine the level of bilirubin in the amniotic fluid. This testing will determine the severity of fetal hemolytic anemia.

Which information should nurses provide to expectant mothers when teaching them how to evaluate daily fetal movement counts (DFMCs)? a. Alcohol or cigarette smoke can irritate the fetus into greater activity. b. Kick counts should be taken every hour and averaged every 6 hours, with every other 6-hour stretch off. c. The fetal alarm signal should go off when fetal movements stop entirely for 12 hours. d. A count of less than four fetal movements in 1 hour warrants future evaluation.

C No movement in a 12-hour period is cause for investigation and possibly intervention. Alcohol and cigarette smoke temporarily reduce fetal movement. The mother should count fetal activity (kick counts) two or three times daily for 60 minutes each time. A count of less than 3 in 1 hour warrants further evaluation by a NST.

A 30-year-old gravida 3, para 2-0-0-2 is at 18 weeks of gestation. Which screening test should the nurse recommend be ordered for this client? a. BPP b. Chorionic villi sampling c. MSAFP screening d. Screening for diabetes mellitus

C The biochemical assessment MSAFP test is performed from week 15 to week 20 of gestation (weeks 16 to 18 are ideal). A BPP is a method of biophysical assessment of fetal well-being in the third trimester. Chorionic villi sampling is a biochemical assessment of the fetus that should be performed from the 10th to 12th weeks of gestation. Screening for diabetes mellitus begins with the first prenatal visit.

A nurse is providing instructions for a nonstress test (NST) to a woman who is at 36 weeks of gestation. Which statement by the client indicates a correct understanding of the nurse's instructions? A) "This test will help to determine whether the baby has Down syndrome or a neural tube defect." B) "I will need to have a full bladder for the test to be done accurately." C) "This test observes for fetal activity and an acceleration of the fetal heart rate to determine the well-being of the baby." D) "I should have my husband drive me home after the test because I may be nauseated."

C) "This test observes for fetal activity and an acceleration of the fetal heart rate to determine the well-being of the baby." The NST is one of the most widely used techniques to determine fetal well-being and is accomplished by monitoring fetal heart rate in conjunction with fetal activity and movements. An ultrasound requires a full bladder. An amniocentesis is the test after which a pregnant woman should be driven home. A maternal serum alpha-fetoprotein test is used in conjunction with unconjugated estriol levels and human chorionic gonadotropin helps to detect Down syndrome.

A group of nurses are discussing the strengths and limitations of various biochemical assessments during pregnancy. Which statement should the nurses indicate as correct? A) Chorionic villus sampling (CVS) is becoming more popular because it provides early diagnosis. B) Percutaneous umbilical blood sampling (PUBS) is one of the quad-screen tests for Down syndrome. C) MSAFP is a screening tool only; it identifies candidates for more definitive procedures. D) Screening for maternal serum alpha-fetoprotein (MSAFP) levels is recommended only for women at risk for neural tube defects.

C) MSAFP is a screening tool only; it identifies candidates for more definitive procedures. CVS does provide a rapid result, but it is declining in popularity because of advances in noninvasive screening techniques. MSAFP screening is recommended for all pregnant women. MSAFP, not PUBS, is part of the quad-screen tests for Down syndrome. MSAFP is a screening tool, not a diagnostic tool. Further diagnostic testing is indicated after an abnormal result.

When helping the mother, father, and other family members actualize the loss of the infant, nurses should: a. Use the words lost or gone rather than dead or died. b. Make sure that the family understands that it is important to name the baby. c. If the parents choose to visit with the baby, apply powder and lotion to the baby and wrap the infant in a pretty blanket. d. Set a firm time for ending the visit with the baby so the parents know when to let go.

C. If the parents choose to visit with the baby, apply powder, and lotion to the baby and wrap the infant in al pretty infant. Presenting the baby in a nice way stimulates the parents' senses and provides pleasant memories of their baby. Nurses must use dead and died to assist the bereaved in accepting reality. Although naming the baby can be helpful, it is important not to create the sense that parents have to name the baby. In fact, some cultural taboos and religious rules prohibit the naming of an infant who has died. Parents need different time periods with their baby to say goodbye. Nurses need to be careful not to rush the process.

Nurses should be aware of the strengths and limitations of various biochemical assessments during pregnancy. Which statement regarding monitoring techniques is the most accurate? a. Chorionic villus sampling (CVS) is becoming more popular because it provides early diagnosis. b. MSAFP screening is recommended only for women at risk for NTDs. c. PUBS is one of the triple-marker tests for Down syndrome. d. MSAFP is a screening tool only; it identifies candidates for more definitive diagnostic procedures.

D MSAFP is a screening tool, not a diagnostic tool. CVS provides a rapid result, but it is declining in popularity because of advances in noninvasive screening techniques. An MSAFP screening is recommended for all pregnant women. MSAFP screening, not PUBS, is part of the triple-marker tests for Down syndrome.

How does the nurse document a NST during which two or more FHR accelerations of 15 beats per minute or more occur with fetal movement in a 20-minute period? a. Nonreactive b. Positive c. Negative d. Reactive

D The NST is reactive (normal) when two or more FHR accelerations of at least 15 beats per minute (each with a duration of at least 15 seconds) occur in a 20-minute period. A nonreactive result means that the heart rate did not accelerate during fetal movement. A positive result is not used with NST. CST uses positive as a result term. A negative result is not used with NST. CST uses negative as a result term.

In comparing the abdominal and transvaginal methods of ultrasound examination, which information should the nurse provide to the client? a. Both require the woman to have a full bladder. b. The abdominal examination is more useful in the first trimester. c. Initially, the transvaginal examination can be painful. d. The transvaginal examination allows pelvic anatomy to be evaluated in greater detail.

D The transvaginal examination allows pelvic anatomy to be evaluated in greater detail than the abdominal method and also allows intrauterine pregnancies to be diagnosed earlier. The abdominal examination requires a full bladder; the transvaginal examination requires an empty one. The transvaginal examination is more useful in the first trimester; the abdominal examination works better after the first trimester. Neither the abdominal nor the transvaginal method of ultrasound examination should be painful, although the woman will feel pressure as the probe is moved during the transvaginal examination.

A woman arrives at the clinic seeking confirmation that she is pregnant. The following information is obtained: She is 24 years old with a body mass index (BMI) of 17.5. She admits to having used cocaine several times during the past year and occasionally drinks alcohol. Her blood pressure is 108/70 mm Hg. The family history is positive for diabetes mellitus and cancer. Her sister recently gave birth to an infant with a neural tube defect (NTD). Which characteristics places this client in a high-risk category? a. Blood pressure, age, BMI b. Drug and alcohol use, age, family history c. Family history, blood pressure (BP), BMI d. Family history, BMI, drug and alcohol abuse

D The womans family history of an NTD, her low BMI, and her drug and alcohol use abuse are high risk factors of pregnancy. The womans BP is normal, and her age does not put her at risk. Her BMI is low and may indicate poor nutritional status, which is a high risk.

A nurse is reviewing categories of high risk pregnancy. Which of the options listed here should the nurse not include? A) Psychosocial B) Environmental C) Biophysical D) Geographic

D) Geographic The fourth category is correctly referred to as the sociodemographic risk category. The factors stem from the mother and her family. Ethnicity may be one of the risks to pregnancy; however, it is not the only factor in this category. Low income, lack of prenatal care, age, parity, and marital status are included. Biophysical is one of the broad categories used for determining risk. It includes genetic considerations, nutritional status, and medical and obstetric disorders. Psychosocial risks include smoking, caffeine, drugs, alcohol, and psychologic status. All of these adverse lifestyles can have a negative effect on the health of the mother or fetus. Environmental risks are those that can affect fertility and fetal development. They include infections, chemicals, radiation, pesticides, illicit drugs, and industrial pollutants.

A nurse is reviewing clinical indications for a contraction stress test(CST). What should the nurse identify as being an appropriate indicator for this test? A) History of preterm labor and intrauterine growth restriction B) Adolescent pregnancy and poor prenatal care C) Increased fetal movement and small for gestational age D) Maternal diabetes mellitus and post-maturity

D) Maternal diabetes mellitus and postmaturity Decreased fetal movement is an indicator for performing a contraction stress test; the size (small for gestational age) is not an indicator. Although adolescent pregnancy and poor prenatal care are risk factors for poor fetal outcomes, they are not indicators for performing a contraction stress test. Intrauterine growth restriction is an indicator; history of a previous stillbirth, not preterm labor, is another indicator.

A family is visiting two surviving triplets. The third triplet died 2 days ago. What action would indicate that the family had begun to grieve for the dead infant? a. They refer to the two live infants as twins. b. They ask about the dead triplet's current status. c. They bring in play clothes for all three infants. d. They refer to the dead infant in the past tense.

D. They refer to the dead infant in the past tense Accepting that the infant is dead (in the past tense of the word) demonstrates acceptance of the reality and that the family has begun to grieve. Referring to the two live infants as twins, asking about the dead infant's current status, and bringing clothing for all three infants indicate that the parents are still in denial.

A woman with severe preeclampsia has been receiving magnesium sulfate by IV infusion for 8 hours. The nurse assesses the woman and documents the following findings: temperature 37.1° C, pulse rate 96 beats/min, respiratory rate 24 breaths/min, blood pressure 155/112 mm Hg, 3+ deep tendon reflexes, and no ankle clonus. The nurse calls the physician, anticipating an order for: Hydralazine. Magnesium sulfate bolus. Diazepam. Calcium gluconate.

Hydralazine Rationale: Hydralazine is an antihypertensive commonly used to treat hypertension in severe preeclampsia. An additional bolus of magnesium sulfate may be ordered for increasing signs of central nervous system irritability related to severe preeclampsia (e.g., clonus) or if eclampsia develops. Diazepam sometimes is used to stop or shorten eclamptic seizures. Calcium gluconate is used as the antidote for magnesium sulfate toxicity. The client is not currently displaying any signs or symptoms of magnesium toxicity.

A nurse caring for pregnant women must be aware that the most common medical complication of pregnancy is: Hypertension. Hyperemesis gravidarum. Hemorrhagic complications. Infections.

Hypertension. Rationale: Preeclampsia and eclampsia are two noted, deadly forms of hypertension, which is the most common medical complication of pregnancy. A large percentage of pregnant women have nausea and vomiting, but a relative few have the severe form called hyperemesis gravidarum. Hemorrhagic complications are the second most common medical complication of pregnancy.

Nurses should be aware that HELLP syndrome: Is a mild form of preeclampsia. Can be diagnosed by a nurse alert to its symptoms. Is characterized by hemolysis, elevated liver enzymes, and low platelets. Is associated with preterm labor but not perinatal mortality.

Is characterized by hemolysis, elevated liver enzymes, and low platelets. Rationale: The acronym HELLP stands for hemolysis (H), elevated liver enzymes (EL), and low platelets (LP). HELLP syndrome is a variant of severe preeclampsia. It is difficult to identify, because the symptoms often are not obvious. It must be diagnosed in the laboratory. Preterm labor is greatly increased with HELLP syndrome, and so is perinatal mortality.

Which laboratory values would be found in a patient diagnosed with preeclampsia? (Select all that apply.) Hemoglobin 8g/dL Platelet count of 75,000 LDH 100 units/L Burr cells BUN 25 mg/dL

Platelet count of 75,000 LDH 100 units/L BUN 25 mg/dL Rationale: Thrombocytopenia below 100,000, an increase in LDH, and an increase in BUN would be noted. Hemoglobin levels would be increased, but 8 g/dL reflects a decreased level. Burr cells would not be present in preeclampsia but would in HELLP syndrome.

A nurse is monitoring a patient's reflexes (DTRs) while receiving magnesium sulfate therapy for treatment of preeclampsia. Which assessment finding indicates a cause for concern? Bilateral DTRs noted at 2+ DTRs response has been noted at 1+ since onset of therapy Positive clonus response elicited unilaterally Patient reports no pain upon examination of DTRs by nurse

Positive clonus response elicited unilaterally Rationale: Positive clonus response elicited unilaterally is a cause for concern as it suggests a hyperactive response. Typically, there is no pain associated with determination of DTRs so this finding would be considered to be normal, as would bilateral DTRs noted at 2+. Even though DTRs at 1+ indicate a sluggish or decreased response, this finding is unchanged since the initiation of therapy. The nurse would continue to monitor.

8. A client asks her nurse, "My doctor told me that he is concerned with the grade of my placenta because I am overdue. What does that mean?" The best response by the nurse is: a. "Your placenta changes as your pregnancy progresses, and it is given a score that indicates the amount of calcium deposits it has. The more calcium deposits, the higher the grade, or number, that is assigned to the placenta. It also means that less blood and oxygen can be delivered to your baby." b. "Your placenta isn't working properly, and your baby is in danger." c. "This means that we will need to perform an amniocentesis to detect if you have any placental damage." d. "Don't worry about it. Everything is fine."

a. "Your placenta changes as your pregnancy progresses, and it is given a score that indicates the amount of calcium deposits it has. The more calcium deposits, the higher the grade, or number, that is assigned to the placenta. It also means that less blood and oxygen can be delivered to your baby."

Early postpartum hemorrhage is defined as a blood loss greater than: a. 500 mL in the first 24 hours after vaginal delivery. b. 750 mL in the first 24 hours after vaginal delivery. c. 1000 mL in the first 48 hours after cesarean delivery. d. 1500 mL in the first 48 hours after cesarean delivery.

a. 500 mL in the first 24 hours after vaginal delivery.

16. Compared with contraction stress test (CST), nonstress test (NST) for antepartum fetal assessment: a. Has no known contraindications. b. Has fewer false-positive results. c. Is more sensitive in detecting fetal compromise. d. Is slightly more expensive.

a. Has no known contraindications.

What PPH conditions are considered medical emergencies that require immediate treatment? a. Inversion of the uterus and hypovolemic shock b. Hypotonic uterus and coagulopathies c. Subinvolution of the uterus and idiopathic thrombocytopenic purpura d. Uterine atony and disseminated intravascular coagulation

a. Inversion of the uterus and hypovolemic shock Inversion of the uterus and hypovolemic shock are considered medical emergencies. Although hypotonic uterus and coagulopathies, subinvolution of the uterus and idiopathic thrombocytopenic purpura, and uterine atony and disseminated intravascular coagulation are serious conditions, they are not necessarily medical emergencies that requires immediate treatment.

13. Nurses should be aware that the biophysical profile (BPP): a. Is an accurate indicator of impending fetal well being. b. Is a compilation of health risk factors of the mother during the later stages of pregnancy. c. Consists of a Doppler blood flow analysis and an amniotic fluid index. d. Involves an invasive form of ultrasound examination.

a. Is an accurate indicator of impending fetal well being.

22. Which analysis of maternal serum may predict chromosomal abnormalities in the fetus? a. Multiple-marker screening b. Lecithin/sphingomyelin (L/S) ratio c. Biophysical profile d. Type and crossmatch of maternal and fetal serum

a. Multiple-marker screening

2. A 39-year-old primigravida thinks that she is about 8 weeks pregnant, although she has had irregular menstrual periods all her life. She has a history of smoking approximately one pack of cigarettes a day, but she tells you that she is trying to cut down. Her laboratory data are within normal limits. What diagnostic technique could be used with this pregnant woman at this time? a. Ultrasound examination b. Maternal serum alpha-fetoprotein (MSAFP) screening c. Amniocentesis d. Nonstress test (NST)

a. Ultrasound examination

The perinatal nurse is caring for a woman in the immediate postbirth period. Assessment reveals that the woman is experiencing profuse bleeding. The most likely etiology for the bleeding is: a. Uterine atony. b. Uterine inversion. c. Vaginal hematoma. d. Vaginal laceration.

a. Uterine atony. Uterine atony is marked hypotonia of the uterus. It is the leading cause of postpartum hemorrhage. Uterine inversion may lead to hemorrhage, but it is not the most likely source of this client's bleeding. Furthermore, if the woman were experiencing a uterine inversion, it would be evidenced by the presence of a large, red, rounded mass protruding from the introitus. A vaginal hematoma may be associated with hemorrhage. However, the most likely clinical finding would be pain, not the presence of profuse bleeding. A vaginal laceration may cause hemorrhage, but it is more likely that profuse bleeding would result from uterine atony. A vaginal laceration should be suspected if vaginal bleeding continues in the presence of a firm, contracted uterine fundus.

How does HELP syndrome occur? What are the pathophysiologic changes?

arteriolar vasospasm, endothelial cell dysfxn with fibrin deposits, and adherence of platelets in blood vessels RBC's damaged as they pass through narrowed blood vessels and become hemolyzed= decreased RBC count and hyperbilirubinemia Endothelial damage and fibrin deposits in liver= impaired liver dysfxn and can lead to hemorrhagic necrosis

Which woman is at greatest risk for early postpartum hemorrhage (PPH)? a. A primiparous woman (G 2 P 1 0 0 1) being prepared for an emergency cesarean birth for fetal distress b. A woman with severe preeclampsia who is receiving magnesium sulfate and whose labor is being induced c. A multiparous woman (G 3 P 2 0 0 2) with an 8-hour labor d. A primigravida in spontaneous labor with preterm twins

b. A woman with severe who is receiving magnesium sulfate and whose labor is being induced. Magnesium sulfate administration during labor poses a risk for PPH. Magnesium acts as a smooth muscle relaxant, thereby contributing to uterine relaxation and atony. Although many causes and risk factors are associated with PPH, the primiparous woman being prepared for an emergency c-section, the multiparous woman with 8-hour labor, and the primigravida in spontaneous labor do not pose risk factors or causes of early PPH.

A woman is diagnosed with having a stillborn. At first, she appears stunned by the news, cries a little, and then asks you to call her mother. The phase of bereavement the woman is experiencing is called: a. Anticipatory grief. b.Acute distress. c. Intense grief. d. Reorganization.

b. Acute distress. The immediate reaction to news of a perinatal loss or infant death encompasses a period of acute distress. Disbelief and denial can occur. However, parents also feel very sad and depressed. Intense outbursts of emotion and crying are normal. However, lack of affect, euphoria, and calmness may occur and may reflect numbness, denial, or personal ways of coping with stress. Anticipatory grief applies to the grief related to a potential loss of an infant. The parent grieves in preparation of the infant's possible death, although he or she clings to the hope that the child will survive. Intense grief occurs in the first few months after the death of the infant. This phase encompasses many different emotions, including loneliness, emptiness, yearning, guilt, anger, and fear. Reorganization occurs after a long and intense search for meaning. Parents are better able to function at work and home, experience a return of self-esteem and confidence, can cope with new challenges, and have placed the loss in perspective.

The prevalence of urinary incontinence (UI) increases as women age, with more than one third of women in the United States suffering from some form of this disorder. The symptoms of mild to moderate UI can be successfully decreased by a number of strategies. Which of these should the nurse instruct the client to use first? a. Pelvic floor support devices b. Bladder training and pelvic muscle exercises c. Surgery d. Medications

b. Bladder training and pelvic muscle exercises Pelvic muscle exercises, known as Kegel exercises, along with bladder training can significantly decrease or entirely relieve stress incontinence in many women. Pelvic floor support devices, also known as pessaries, come in a variety of shapes and sizes. Pessaries may not be effective for all women and require scrupulous cleaning to prevent infection. Anterior and posterior repairs and even a hysterectomy may be performed. If surgical repair is performed, the nurse must focus her care on preventing infection and helping the woman avoid putting stress on the surgical site. Pharmacologic therapy includes serotonin-norepinephrine uptake inhibitors or vaginal estrogen therapy. These are not the first action a nurse should recommend.

A mother in late middle age who is certain she is not pregnant tells the nurse during an office visit that she has urinary problems and sensations of bearing down and of something in her vagina. The nurse would realize that the client most likely is suffering from: a. Pelvic relaxation. b. Cystoceles and/or rectoceles. c. Uterine displacement. d. Genital fistulas.

b. Cystoceles and/or rectoceles. Cystoceles are protrusions of the bladder downward into the vagina; rectoceles are herniations of the anterior rectal wall through a relaxed or ruptured vaginal fascia. Both can present as a bearing down sensation with urinary dysfunction. They occur more often in older women who have borne children.

18. A woman has been diagnosed with a high risk pregnancy. She and her husband come into the office in a very anxious state. She seems to be coping by withdrawing from the discussion, showing declining interest. The nurse can best help the couple by: a. Telling her that the physician will isolate the problem with more tests. b. Encouraging her and urging her to continue with childbirth classes. c. Becoming assertive and laying out the decisions the couple needs to make. d. Downplaying her risks by citing success rate studies.

b. Encouraging her and urging her to continue with childbirth classes.

24. Which nursing intervention is necessary before a second-trimester transabdominal ultrasound? a. Place the woman NPO for 12 hours. b. Instruct the woman to drink 1 to 2 quarts of water. c. Administer an enema. d. Perform an abdominal preparation.

b. Instruct the woman to drink 1 to 2 quarts of water.

During a follow-up visit, if parents have progressed to the second stage or phase of grieving, the nurse should not expect to see: a. Guilt, particularly in the mother. b. Numbness or lack of response. c. Bitterness or irritability. d. Fear and anxiety, especially about getting pregnant again.

b. Numbness or lack of response The second phase of grieving encompasses a wide range of intense emotions, including guilt, anger, bitterness, fear, and anxiety. What the nurse would hope not to see is numbness or unresponsiveness, as if the parents were still in denial or shock.

A primary nursing responsibility when caring for a woman experiencing an obstetric hemorrhage associated with uterine atony is to: a. Establish venous access. b. Perform fundal massage. c. Prepare the woman for surgical intervention. d. Catheterize the bladder.

b. Perform fundal massage. The initial management of excessive postpartum bleeding is firm massage of the uterine fundus. Although establishing venous access may be a necessary intervention, the initial intervention would be fundal massage. The woman may need surgical intervention to treat her postpartum hemorrhage, but the initial nursing intervention would be to assess the uterus. After uterine massage the nurse may want to catheterize the client to eliminate any bladder distention that may be preventing the uterus from contracting properly.

20. Risk factors tend to be interrelated and cumulative in their effect. While planning the care for a laboring client with diabetes mellitus, the nurse is aware that she is at a greater risk for: a. Oligohydramnios. b. Polyhydramnios. c. Postterm pregnancy. d. Chromosomal abnormalities.

b. Polyhydramnios.

The perinatal nurse assisting with establishing lactation is aware that acute mastitis can be minimized by: a. Washing the nipples and breasts with mild soap and water once a day. b. Using proper breastfeeding techniques. c. Wearing a nipple shield for the first few days of breastfeeding. d. Wearing a supportive bra 24 hours a day.

b. Using proper breastfeeding techniques. Almost all instances of acute mastitis can be avoided by proper breastfeeding technique to prevent cracked nipples. Washing the nipples and breasts daily is no longer indicated. In fact, this can cause tissue dryness and irritation, which can lead to tissue breakdown and infection. Wearing a nipple shield does not prevent mastitis. Wearing a supportive bra 24 hours a day may contribute to mastitis, especially if an underwire bra is worn, because it may put pressure on the upper, outer area of the breast, contributing to blocked ducts and mastitis.

Despite popular belief, there is a rare type of hemophilia that affects women of childbearing age. von Willebrand disease is the most common of the hereditary bleeding disorders and can affect males and females alike. It results from a factor VIII deficiency and platelet dysfunction. Although factor VIII levels increase naturally during pregnancy, there is an increased risk for postpartum hemorrhage from birth until 4 weeks after delivery as levels of von Willebrand factor (vWf) and factor VIII decrease. The treatment that should be considered first for the client with von Willebrand disease who experiences a postpartum hemorrhage is: a. Cryoprecipitate. b.Factor VIII and vWf. c. Desmopressin d. Hemabate.

c. Desmopressin Desmopressin is the primary treatment of choice. This hormone can be administered orally, nasally, and intravenously. This medication promotes the release of factor VIII and vWf from storage. Cryoprecipitate may be used; however, because of the risk of possible donor viruses, other modalities are considered safer. Treatment with plasma products such as factor VIII and vWf are an acceptable option for this client. Because of the repeated exposure to donor blood products and possible viruses, this is not the initial treatment of choice. Although the administration of this prostaglandin is known to promote contraction of the uterus during postpartum hemorrhage, it is not effective for the client who presents with a bleeding disorder.

When a woman is diagnosed with postpartum depression (PPD) with psychotic features, one of the main concerns is that she may: a. Have outbursts of anger. b. Neglect her hygiene. c. Harm her infant. d. Lose interest in her husband.

c. Harm her infant. Thoughts of harm to one's self or the infant are among the most serious symptoms of PPD and require immediate assessment and intervention. Although outbursts of anger, hygiene neglect, and loss of interest in her husband are attributable to PPD, the major concern would be the potential to harm herself or her infant.

17. The nurse providing care for the antepartum woman should understand that contraction stress test (CST): a. Sometimes uses vibroacoustic stimulation. b. Is an invasive test; however, contractions are stimulated. c. Is considered negative if no late decelerations are observed with the contractions. d. Is more effective than nonstress test (NST) if the membranes have already been ruptured.

c. Is considered negative if no late decelerations are observed with the contractions.

To provide adequate postpartum care, the nurse should be aware that postpartum depression (PPD) without psychotic features: a. Means that the woman is experiencing the baby blues. In addition she has a visit with a counselor or psychologist. b. Is more common among older, Caucasian women because they have higher expectations. c. Is distinguished by irritability, severe anxiety, and panic attacks. d. Will disappear on its own without outside help.

c. Is distinguished by irritability, severe anxiety, and panic attacks. PPD is also characterized by spontaneous crying long after the usual duration of the baby blues. PPD, even without psychotic features, is more serious and persistent than postpartum baby blues. It is more common among younger mothers and African-American mothers. Most women need professional help to get through PPD, including pharmacologic intervention.

During the initial acute distress phase of grieving, parents still must make unexpected and unwanted decisions about funeral arrangements and even naming the baby. The nurse's role should be to: a. Take over as much as possible to relieve the pressure. b. Encourage grandparents to take over. c. Make sure the parents themselves approve the final decisions. d. Let them alone to work things out.

c. Make sure the parents themselves approve the final decisions. The nurse is always the client's advocate. Nurses can offer support and guidance and leave room for the same from grandparents. However, in the end nurses should strive to let the parents make the final decisions.

What infection is contracted mostly by first-time mothers who are breastfeeding? a. Endometritis b. Wound infections c. Mastitis d. Urinary tract infections

c. Mastitis Mastitis is infection in a breast, usually confined to a milk duct. Most women who suffer this are first-timers who are breastfeeding.

The first and most important nursing intervention when a nurse observes profuse postpartum bleeding is to: a. Call the woman's primary health care provider. b. Administer the standing order for an oxytocic. c. Palpate the uterus and massage it if it is boggy. d. Assess maternal blood pressure and pulse for signs of hypovolemic shock.

c. Palpate the uterus and massage it if it is boggy. The initial management of excessive postpartum bleeding is firm massage of the uterine fundus. Though calling the health care provider, administering an oxytocic, and assessing maternal BP are appropriate interventions, the primary intervention should be to assess the uterus. Uterine atony is the leading cause of postpartum hemorrhage (PPH).

Anxiety disorders are the most common mental disorders that affect women. While providing care to the maternity patient, the nurse should be aware that one of these disorders is likely to be triggered by the process of labor and birth. This disorder is: a. Phobias. b. Panic disorder. c. Post-traumatic stress disorder (PTSD). d. Obsessive-compulsive disorder (OCD).

c. Post-traumatic stress disorder (PTSD).

A mother with mastitis is concerned about breastfeeding while she has an active infection. The nurse should explain that: a. The infant is protected from infection by immunoglobulins in the breast milk. b. The infant is not susceptible to the organisms that cause mastitis c. The organisms that cause mastitis are not passed to the milk. d. The organisms will be inactivated by gastric acid.

c. The organisms that cause mastitis are not passed to the milk.

A woman who has recently given birth complains of pain and tenderness in her leg. On physical examination the nurse notices warmth and redness over an enlarged, hardened area. The nurse should suspect __________ and should confirm the diagnosis by ___________. a. Disseminated intravascular coagulation; asking for laboratory tests b. von Willebrand disease; noting whether bleeding times have been extended c. Thrombophlebitis; using real-time and color Doppler ultrasound d. Coagulopathies; drawing blood for laboratory analysis

c. Thrombophlebitis; using real-time and color Doppler ultrasound Pain and tenderness in the extremities, which show warmth, redness, and hardness, likely indicate thrombophlebitis. A Doppler ultrasound is a common noninvasive way to confirm diagnosis.

After giving birth to a stillborn infant, the woman turns to the nurse and says, "I just finished painting the baby's room. Do you think that caused my baby to die?" The nurse's best response to this woman is: a. "That's an old wives' tale; lots of women are around paint during pregnancy, and this doesn't happen to them." b. "That's not likely. Paint is associated with elevated pediatric lead levels." c. Silence. d. "I can understand your need to find an answer to what caused this. What else are you thinking about?"

d. "I can understand your need to find an answer to what caused this. What else are you thinking about?" The statement, "I can understand your need to find an answer to what caused this. What else are you thinking about?" is very appropriate for the nurse. It demonstrates caring and compassion and allows the mother to vent her thoughts and feelings, which is therapeutic in the process of grieving. The nurse should resist the temptation to give advice or to use clichés in offering support to the bereaved. In addition, trying to give bereaved parents answers when no clear answers exist or trying to squelch their guilt feeling does not help the process of grief. Trying to give bereaved parents answers when no clear answers exist does not help the grief process. In addition, this response probably would increase the mother's feelings of guilt. One of the most important goals of the nurse is to validate the experience and feelings of the parents by encouraging them to tell their stories and listening with care. The nurse should encourage the mother to express her ideas.

A woman experienced a miscarriage at 10 weeks of gestation and had a dilation and curettage (D&C). She states that she is just fine and wants to go home as soon as possible. While you are assessing her responses to her loss, she tells you that she had purchased some baby things and had picked out a name. On the basis of your assessment of her responses, what nursing intervention would you use first? a. Ready her for discharge. b. Notify pastoral care to offer her a blessing. c. Ask her whether she would like to see what was obtained from her D&C. d. Ask her what name she had picked out for her baby.

d. Ask her what name she had picked out for her baby. One way of actualizing the loss is to allow parents to name the infant. The nurse should follow this client's cues and inquire about naming the infant. The client is looking for an opportunity to express her feelings of loss. The nurse should take this opportunity to offer support by allowing the woman to talk about her feelings. Furthermore, one way of actualizing the loss is to allow parents to name the infant. The nurse should follow this client's cues and inquire about naming the infant. Although it may be therapeutic to offer religious support, the nurse should take this opportunity to offer support by allowing the woman to talk about her feelings. Furthermore, one way of actualizing the loss is to allow parents to name the infant. Asking the woman if she would like to see what was obtained from her D&C is completely inappropriate.

If nonsurgical treatment for late postpartum hemorrhage is ineffective, which surgical procedure is appropriate to correct the cause of this condition? a. Hysterectomy b. Laparoscopy c. Laparotomy d. D&C

d. D&C

1. A woman arrives at the clinic seeking confirmation that she is pregnant. The following information is obtained: She is 24 years old with a body mass index (BMI) of 17.5. She admits to having used cocaine "several times" during the past year and drinks alcohol occasionally. Her blood pressure (BP) is 108/70 mm Hg, her pulse rate is 72 beats/min, and her respiratory rate is 16 breaths/min. The family history is positive for diabetes mellitus and cancer. Her sister recently gave birth to an infant with a neural tube defect (NTD). Which characteristics place the woman in a high risk category? a. Blood pressure, age, BMI b. Drug/alcohol use, age, family history c. Family history, blood pressure, BMI d. Family history, BMI, drug/alcohol abuse

d. Family history, BMI, drug/alcohol abuse

The nurse caring for a family during a loss may notice that survival guilt sometimes is felt at the death of an infant by the child's: a. Siblings. b. Mother. c. Father. d. Grandparents.

d. Grandparents. Survival guilt sometimes is felt by grandparents, because they believe that the death upsets the natural order of things. They are angry that they are alive and their grandchild is not.

The most appropriate statement that the nurse can make to bereaved parents is: a. "You have an angel in heaven." b. "I understand how you must feel." c. "You're young and can have other children." d. "I'm sorry."

d. I'm sorry One of nurse's most important goals is to validate the experience and feelings of the parents by encouraging them to tell their stories and listening with care. At the very least, the nurse should acknowledge the loss with a simple but sincere comment such as, "I'm sorry." The initial impulse may be to reduce one's sense of helplessness and say or do something that you think will reduce their pain. Although such a response may seem supportive at the time, it can stifle the further expression of emotion. The nurse should resist the temptation to give advice or to use clichés when offering support to the bereaved. Saying, "You're young and can have other children" is not a therapeutic response for the nurse to make.

15. Nurses should be aware of the strengths and limitations of various biochemical assessments during pregnancy, including that: a. Chorionic villus sampling (CVS) is becoming more popular because it provides early diagnosis. b. Maternal serum alpha-fetoprotein (MSAFP) screening is recommended only for women at risk for neural tube defects. c. Percutaneous umbilical blood sampling (PUBS) is one of the triple-marker tests for Down syndrome. d. MSAFP is a screening tool only; it identifies candidates for more definitive procedures.

d. MSAFP is a screening tool only; it identifies candidates for more definitive procedures.

To provide adequate postpartum care, the nurse should be aware that postpartum depression (PPD) with psychotic features: a. Is more likely to occur in women with more than two children. b. Is rarely delusional and then is usually about someone trying to harm her (the mother). c. Although serious, is not likely to need psychiatric hospitalization. d. May include bipolar disorder (formerly called "manic depression").

d. May include bipolar disorder (formerly called "manic depression"). Manic mood swings are possible. PPD is more likely to occur in first-time mothers. Delusions may be present in 50% of women with PPD, usually about something being wrong with the infant. PPD with psychosis is a psychiatric emergency that requires hospitalization.

Which condition is a transient, self-limiting mood disorder that affects new mothers after childbirth? a. Postpartum depression b.Postpartum psychosis c. Postpartum bipolar disorder d. Postpartum blues

d. Postpartum blues

25. The nurse recognizes that a nonstress test (NST) in which two or more fetal heart rate (FHR) accelerations of 15 beats/min or more occur with fetal movement in a 20-minute period is: a. Nonreactive b. Positive c. Negative d. Reactive

d. Reactive

With shortened hospital stays, new mothers are often discharged before they begin to experience symptoms of the baby blues or postpartum depression. As part of the discharge teaching, the nurse can prepare the mother for this adjustment to her new role by instructing her regarding self-care activities to help prevent postpartum depression. The most accurate statement as related to these activities is to: a. Stay home and avoid outside activities to ensure adequate rest. b. Be certain that you are the only caregiver for your baby, to facilitate infant attachment. c. Keep feelings of sadness and adjustment to your new role to yourself. d. Realize that this is a common occurrence that affects many women

d. Realize that this is a common occurrence that affects many women Should the new mother experience symptoms of the baby blues, it is important that she be aware that this is nothing to be ashamed of. As many as 15% of new mothers experience similar symptoms. Although it is important for the mother to obtain enough rest, she should not distance herself from family and friends. Her spouse or partner can communicate the best visiting times so the new mother can obtain adequate rest. It is also important that she not isolate herself at home during this time of role adjustment. Even if breastfeeding, other family members can participate in the infant's care. If depression occurs, the symptoms can often interfere with mothering functions and this support will be essential. The new mother should share her feelings with someone else. It is also important that she not overcommit herself or think she has to be "superwoman." A telephone call to the hospital warm line may provide reassurance with lactation issues and other infant care questions. Should symptoms continue, a referral to a professional therapist may be necessary.

Nurses need to know the basic definitions and incidence data about postpartum hemorrhage (PPH). For instance: a. PPH is easy to recognize early; after all, the woman is bleeding. b. Traditionally it takes more than 1000 mL of blood after vaginal birth and 2500 mL after cesarean birth to define the condition as PPH. c. If anything, nurses and doctors tend to overestimate the amount of blood loss. d. Traditionally PPH has been classified as early or late with respect to birth.

d. Traditionally PPH has been classified as early or late with respect to birth. Early PPH is also known as primary, or acute, PPH; late PPH is known as secondary PPH. Unfortunately PPH can occur with little warning and often is recognized only after the mother has profound symptoms. Traditionally a 500-ml blood loss after a vaginal birth and a 1000-ml blood loss after a cesarean birth constitute PPH. Medical personnel tend to underestimate blood loss by as much as 50% in their subjective observations.

When caring for a postpartum woman experiencing hemorrhagic shock, the nurse recognizes that the most objective and least invasive assessment of adequate organ perfusion and oxygenation is: a. Absence of cyanosis in the buccal mucosa. b. Cool, dry skin c. Diminished restlessness. d. Urinary output of at least 30 mL/hr.

d. Urinary output of at least 30 mL/hr. Hemorrhage may result in hemorrhagic shock. Shock is an emergency situation in which the perfusion of body organs may become severely compromised and death may occur. The presence of adequate urinary output indicates adequate tissue perfusion. The assessment of the buccal mucosa for cyanosis can be subjective in nature. The presence of cool, pale, clammy skin would be an indicative finding associated with hemorrhagic shock. Hemorrhagic shock is associated with lethargy, not restlessness.

Define eclampsia

the onset of seizure activity or coma in a woman with preeclampsia who has no hx of preexisting patho resulting in seizure activity


Set pelajaran terkait

Behavior modification - chapter 8

View Set

Chapter 26: Nursing Care of the Child With an Alteration in Metabolism/Endocrine Disorder

View Set

Chapter 1 & 2: Contemporary Nursing

View Set

Pathophysiology - Practice Exam Reproductive

View Set

Thinkful Data Structures and Algorithms

View Set

FIN320F Exam 2 - Smart Book Shannon_Dyer66, Finance Unit 10 - Unit 15, FIN 320F Exam 2 Vocabulary heatherkray, Finance 320F Final Exam Terms JMonroe55, FIN 320F Final (Units 10-15) KirstenTerry23

View Set